You are on page 1of 102

ACKNOWLEDGEMENT

The time and effort provided by the following individuals who served as members of this
committee are greatly appreciated:
James VanRhee, MS, PA-C. Project Director
Linda Allison, MPH, MD
Mark Archambault, MHS, RPA-C
Petar Breitinger, MPAS, PA-C
Christine Bruce, MHSA, PA-C
Ralph Rice, MPAS, PA-C
Eric Vangsnes, MSA, PA-C
Donna Yeisley, Med, PA-C

DEDICATION
This examination would not have been possible without the years of commitment of the MR. TIB
Development Committee. Numerous PA educators from across the nation provided their
experience and insight as questions for MR. TIB. It has been this data bank that served as the
building blocks for PACKRAT.
APAP is proud to be able to continue in the tradition of quality fostered by the forerunners of the
self-assessment examination for physical assistants. It is our honor to dedicate PACKRAT to:
Jesse C. Edwards, MS
Claire S. Parker, PhD
University of Nebraska, Physician Assistant Program

ASSOCIATION OF PHYSICIAN ASSISTANT PROGRAMS


Physician Assistant Clinical Knowledge Rating and Assessment Tool
(PACKRAT)
Form 9
Directions and Explanations

TABLE OF CONTENTS

I.

Introduction

II.

Explanation of the Score Report

Page 1: Scores Your total Score and Group Comparisons


Page 2: Your strengths, Weaknesses, and Quality of Responses
Page 3: Your Individual and Correct Responses
Page 4: Your Responses by Task and Category
Page 5-6: Your Profile Comparison: Demographic Profile

2
2
2
2
2

III.

Recommendations for Using the Feedback Package

IV.

Study Resources

V.

Answer Key

VI.

Examination Explanations

VII.

Comment Form

101

Copyright 2004. Association of Physician Assistant Programs. All rights reserved. No part of this publication may be
reproduced or transmitted in any form or by any means, electronic or mechanical, including photocopy or recording, or
any information and retrieval system, without permission in writing from the Association of Physician Assistant Programs.

PHYSICIAN ASSISTANT CLINICAL KNOWLEDGE RATING


AND ASSESSMENT TOOL (PACKRAT)

I. Introduction
The Physician Assistant Clinical Knowledge Rating and Assessment Tool (PACKRAT) was
developed by a volunteer committee of experts and is based on the content outline of a nationally
recognized competency examination. The following is a description of the content of PACKRAT:

PACKRAT EXAMINATION MATRIX


PACKRAT EXAMINATION MATRIX
CONTENT AREA
NUMBER OF ITEMS
CONTENT AREA
NUMBER OF ITEMS
1. History & Physical
36
1. History
& Physical
26 32
2. Diagnostic
Studies
2. Diagnostic
Studies
36 41
3. Diagnosis
3. Diagnosis
45 22
4. Health Maintenance
4. Health
Maintenance
22 31
5. Clinical
Intervention
5. Clinical
Intervention
32 41
6. Clinical
Therapeutics
6. Clinical
Therapeutics
48 22
7. Scientific
Concepts
7. Scientific Concepts
16
TOTALS
225
TOTALS
225
Additionally, questions also apply to the following clinical specialties:
Additionally, questions also apply to the following clinical specialties:
A. Cardiology
I. Neurology
A. Cardiology
Neurology
B. Dermatology
J. I.
Obstetrics/Gynecology
B. Dermatology
J.
Obstetrics/Gynecology
C. Endocrinology
K. Orthopedics/Rheumatology
C. Endocrinology
Orthopedics/Rheumatology
D. ENT/Ophthalmology
M.K.
Psychiatry/Behavioral
Medicine
D. ENT
Pediatrics
F. Gastrointestinal/Nutritional
N. L.
Pulmonology
E. Ophthalmology
Psychiatry/Behavioral Medicine
H. Hermatology
P. M.
Urology/Renal
F. Gastrointestinal/Nutritional
Pulmonology
Q.N.
Infectious
Diseases
G. Geriatrics
O. Surgery
H. Hematology
P. Urology/Renal

The task and specialty categories for each item are listed in the answer key on page 5; your
feedback package contains a breakdown of responses by the task and clinical specialty category.
Pay particular attention to the questions you answered incorrectly and determine the specialty for
that question and use this information to identify weaknesses.
The PACKRAT provides a detailed feedback report of performance and it is available to anyone
at any time. Explanations were developed for all the questions to provide a rationale for correct,
as well as incorrect, answers. This information will help determine strengths and weaknesses with
respect to the PACKRAT content outline. If you have weaknesses in specific areas, you may
need to obtain additional clinical experience in those areas.
This booklet is designed to explain and interpret the information contained in the accompanying
computerized score report. You can use the report package to learn more about your abilities.

II. Explanation of the Score Report


This section provides an interpretation of each
page of the computerized score report you
received. You should have your computer
score report in front of you. Begin on page 1
of the report and read the following
information.
Page 1: Scores Your Total Score and
Group Comparisons
Page 1 is an overview of the PACKRAT
feedback report. Toward the bottom of the
page is your examination score. This score
shows the number of questions you answered
correctly out of a possible 225. The average
score for all first-year and second-year
candidates who have taken the PACKRAT to
date is also given.
Page 2: Your Strengths, Weaknesses, and
Quality of Responses
Page 2 of the score report gives an overview
of the content area in which your performance
is categorized as Strong, Satisfactory, or
Needing Improvement. These areas are
based on the examination matrix on page 1.
In each content area, your answers have
been classified as correct, acceptable,
unsatisfactory, or harmful. A definition of
these classifications is also provided on this
page. Pay particular attention to the areas
under Needing Improvement, as these areas
should be noted for further study. Also check
the answer key for the specialty area of these
items. If you selected a harmful answer in any
content area, it will be automatically placed in
the
Needing
Improvement
category,
regardless of the number of correct answers
selected. Carefully review these questions
and their explanations and specialty
classifications in Section VI to help you
understand why your answers were correct.
Page 3: Your Individual
Responses

and

Correct

Page 3 lists your answers to all questions.


When your answer differs from the correct
one, the proper response appears in
parentheses. Use Section VI with this page to
review the rationale for each option that is

provided in the explanations, which are


referenced to the study resources. The
explanations may help you understand why
one answer is more appropriate than another,
or not the best answer, and why some of your
answers may have been incorrect. If the
option you chose was judged potentially
harmful to the patient or others, an asterisk (*)
appears before your answer. Options
classified as potentially harmful may identify
serious weaknesses. Go over these questions
carefully and read the explanations for the
correct answers. You may be able to identify
areas where you need further study.
Page 4: Your Responses by Task and
Specialty Category
Page 4 lists your responses by both specialty
and task category. You will be bale to identify
the areas of the content outline where you
may have difficulty. The numbers reflect how
many items you answered correctly out of the
total possible correct within each task and
specialty area. Categories 1 through 7 identify
the task areas and A-P the clinically specialty
areas. If you missed a significant number of
items in an area, check the key and go over
the explanations for the items in these areas.
Page 5-6: Your Profile
Demographic Profile

Comparison:

Page 5 is the beginning of the individual


Demographic Profile Comparison. This profile
shows your reported demographic information
and compares your information to the entire
group of individuals who have taken the
PACKRAT to date. The demographic data are
based on the information you provided by
answering the questions in the test booklet.
The summary demographic information
shown reflects all the data compiled for either
first or second-year students who taken the
PACKRAT. For example, if you are a secondyear physician assistant student, your profile
is compared to all other second year students.
The example shown later indicated this
individual is a second-year student who has 36 months of clinical experience. The profile
also explains that 15% of those who took the
PACKRAT also had 3-6 months experience,
compared to 45% with 10 to 12 months
experience.

RESPONDENT DEMPGRAPHIC
INFORMATION
C. Number of months of clinical rotations
completed at the time of this exam?

Your response:

(
(
(
(
(
(

)
)
)
)
)
)

1.
2.
3.
4.
5.
6.

All Second-year
Respondents:

None
Less than 3 months
3 to 6 months
7 to 9 months
10 to 12 months
Greater than 12 months

( 0% )
( 5% )
( 15%)
( 20%)
(45%)
(15%)

You can use the profile to compare your


characteristics to all those second year
students who have taken the PACKRAT. If
you are a first year student, your profile will be
compared to all first-year students.
III. Recommendations for using The
Feedback Package
As a current physician assistant student,
PACKRAT can be a useful self-evaluation
tool. Through careful review of question
explanations, noting specific tasks and
content areas, you will be able to assess your
current strengths and weaknesses. You will
be able to identify particular areas in which to
concentrate more effort as you continue your
studies. By concentrating your effort on the
areas in which you did not do well, you may
improve your performance, and you may have
a better chance of passing the proctored
examination.
However,
APAP
cannot
guarantee that this will occur, since the
conditions under which you attempted the
PACKRAT may have been different from
those in a standardized administration of a
proctored examination.

perhaps further study would make you more


familiar with these situations.
Once you have completely reviewed your
score report and this booklet, APAP hopes
you will use this information to improve your
overall performance, either on the job or on
future certification examinations. Should you
wish to provide suggestions about PACKRAT
to APAP, you will find a comment form on the
last page of this booklet.
IV. Study Resources
A variety of textbooks are currently available
to assist candidates in preparing for the
certification examination. For additional
information, you may contact a faculty
member at an educational program or an
experienced colleague if you need help
determining which references to review in a
specific content area. A short list of general
textbooks is below. All examination questions
are related to material found in these
resources.
Please note that the books on this list are not
available from APAP. This is not intended as
an all-inclusive list, and the materials listed
below are suggested study materials only.
1.

2.

3.

4.

5.
Use the explanations in Section VI to analyze
why you chose various options. Again, pay
particular attention to the options that were
judged potentially harmful or unsatisfactory.
Look at the question and the four options
again to see why the answer you chose was
incorrect. If there appears to be a deficit in
your exposure to a particular clinical specialty,

6.

7.

Ahya SN, Flood K, and Paranjothi S


(eds). The Washington Manual of
Medical Therapeutics. 30th ed.
Philadelphia, PA: Williams &
Wilkins, 2001.
Andreoli TE, et al (eds). Cecils
Essentials of Medicine. 5th ed.
Philadelphia, PA: WB Saunders
Co., 2001.
Ballweg R et al. Physician Assistant:
rd
A Guide to Clinical Practice. 3 ed.,
Saunders, 2003.
Bates B. Guide to Physical
Examination and History Taking.
8th ed. Philadelphia, PA: JB
Lippincott Co., 2000.
Beckmann CR, et al. Obstetrics &
Gynecology. 4th ed. Philadelphia,
PA: Lippincott Co., 2002.
Behrman RE, et al. Nelsons
Textbook of Pediatrics. 16th ed.
Philadelphia, PA: WB Saunders Co.,
2000.
Berkowitz, C. Pediatrics: A Primary
Care Approach, 2nd ed.

8.

9.

10.

11.

12.

13.

14.

15.

16.

17.

18.

19.

20.

21.

Philadelphia, PA: WB Saunders,


2000.
Braunwald E, et al (eds). Harrisons
Principles of Internal Medicine. 15th
ed. New York, NY: McGraw-Hill,
Inc., 2001.
DeCherney AH & Pernoll ML (eds.)
Current Obstetric & Gynecological
th
Diagnosis & Treatment, 9 ed.,
Norwalk, CT: Appleton & Lange,
2003
Ellsworth AJ, et al. (eds). Mosbys
Medical Drug Reference. St. Louis,
MO: Mosby Yearbook, Inc., 2003.
Fitzpatrick TB, Palano MK, and
Surmond, D. Color Atlas and
Synopsis of Clinical Dermatology.
5th ed. New York, NY: McGraw-Hill,
Inc., 2001.
Goldman J and Bennet JC. Cecil
Textbook of Medicine. 21st.
Philadelphia, PA: WB Saunders
Co., 2000.
Hacker NF and Moore GJ.
Essentials of Obstetrics and
Gynecology. 3rd ed. Philadelphia,
PA: WB Saunders Co., 1998.
Hay WW, et al. Current Pediatric
Diagnosis and Treatment. 16th ed.
Norwalk, CT: Appleton & Lange,
2003.
Kaplan HI and Sadock BJ (eds).
Concise Textbook of Clinical
Psychiatry. Philadelphia, PA:
Williams & Wilkins, 1998.
Katzung BG. Basic and Clinical
Pharmacology. 8th ed. Stamford,
CT: Appleton & Lange, 2001.
Mandel GL, Bennett JE, and Dolin
R. Principles and Practice of
Infectious Disease. 5th ed.,
Churchill Livingston, 2000.
McPhee SJ, et al. Pathophysiology
rd
of Disease. 3 ed., Mcgraw Hill,
2000.
Mercier LR, et al. Practical
Orthopedics. 5th ed. St. Louis, MO:
Mosby Yearbook, Inc., 2000.
Mettler FA, et al. Primary Care
Radiology. Philadelphia, PA: WB
Saunders, Co., 2000.
Mycek MJ, Harvey RA, and Champe
PC. Lippincotts Illustrated Reviews:
Pharmacology. 2nd ed. Baltimore,
MD: Williams & Wilkins, 2000.

22.

23.

24.

25.

26.

27.

28.

29.

30.

31.

32.

Noble J, et al. Textbook of Primary


Care Medicine. 2nd ed. St. Louis,
MO: Mosby Yearbook, Inc., 1996.
Sacher RA and McPherson RA.
Widmann's Clinical Interpretation of
Laboratory Tests. 11th ed. FA
Davis Co., 2000.
Schwartz SI, et al. Principles of
Surgery. 7th ed. New York, NY:
McGraw-Hill, Inc., 1998.
Skinner HB (ed.) Current Diagnosis
nd
& Treatment in Orthopedics. 2 ed.,
Norwalk,CT:Appleton & Lange,
2000.
Steinberg GG. Orthopedics in
rd
Primary Care. 3 ed. Philadelphia,
PA: Lippincott Williams & Wilkins,
rd
3 ed, 1999.
Tierney LM, et al. Current Medical
Diagnosis and Treatment. 42nd ed.
Stamford, CT: Appleton & Lange,
2003.
Tintinalli JE, Krome RL, and Ruiz E.
Emergency Medicine: A
Comprehensive Guide. 5th ed.
New York, NY: McGraw-Hill, Inc.,
2000.
Townsend CM. Sabistons
Textbook of Surgery. The Biological
Basis of Modern Surgical Practice.
16th ed. Philadelphia, PA: WB
Saunders, Co., 2002.
Vaughn D. et al. General
th
Ophthalmology. 15 ed., McGraw
Hill, 1998
Way LW, et al. Current Surgical
th
Diagnosis and Treatment. 11 ed.,
McGraw Hill, 2002.
Wilson WR. Current Diagnosis and
Treatment in Infectious Disease.
Norwalk, CT: Appleton & Lange,
2001.

THIS PAGE INTENTIONALLY LEFT BLANK


FOR ANSWER KEY

ASSOCIATION OF PHYSICIAN ASSISTANT PROGRAMS


Physician Assistant Clinical Knowledge Rating and Assessment Tool
(PACKRAT) Form 9
EXPLANATIONS
1. History & Physical/Cardiology
Which of the following conditions would cause a positive Kussmaul's sign on physical
examination?
Answers
A. Left ventricular failure
B. Pulmonary edema
C. Coarctation of the aorta
D. Constrictive pericarditis
Explanations
(u) A. Left ventricular failure results in the back-up of blood into the left atrium and then the
pulmonary system so it would not be associated with Kussmaul's sign.
(u) B. Pulmonary edema primarily results in increased pulmonary pressures rather than having
effects on the venous inflow into the heart.
(u) C. Coarctation of the aorta primarily affects outflow from the heart due to the stenosis resulting
in delayed and decreased femoral pulses; it has no effect on causing Kussmaul's sign.
(c) D. Kussmaul's sign is an increase rather than the normal decrease in the CVP during
inspiration. It is most often caused by severe right-sided heart failure; it is a frequent finding in
patients with constrictive pericarditis or right ventricular infarction.
Ref: (8)

2. History & Physical/Cardiology


Anginal chest pain is most commonly described as which of the following?
Answers
A. Pain changing with position or respiration
B. A sensation of discomfort
C. Tearing pain radiating to the back
D. Pain lasting for several hours
Explanations
(u) A. Pain changing with position or respiration is suggestive of pericarditis.
(c) B. Myocardial ischemia is often experienced as a sensation of discomfort lasting 5-15 minutes,
described as dull, aching or pressure.
(u) C. Tearing pain with radiation to the back represents aortic dissection.
(u) D. Chest pain lasting for several hours is more suggestive for myocardial infarction.
Ref: (27)

3. History & Physical/Cardiology


Eliciting a history from a patient presenting with dyspnea due to early heart failure, the
severity of the dyspnea should be quantified by
Answers
A. amount of activity that precipitates it.
B. how many pillows they sleep on at night.
C. how long it takes the dyspnea to resolve.
D. any associated comorbidities.
Explanations
(c) A. The amount of activity that precipitates dyspnea should be quantified in the history.
(u) B. Orthopnea or paroxysmal nocturnal dyspnea can be quantified by how many pillows a
patient needs to sleep on to be comfortable.
(u) C. How long dyspnea takes to resolve or associated comorbidities has no bearing on
quantifying the severity of dyspnea.
(u) D. See answer C above.
Ref: (8)

4. History & Physical/Cardiology


A 25 year-old female presents with a three-day history of chest pain aggravated by
coughing and relieved by sitting. She is febrile and a CBC with differential reveals
leukocytosis. Which of the following physical exam signs is characteristic of her problem?
Answers
A. Pulsus paradoxus
B. Localized crackles
C. Pericardial friction rub
D. Wheezing
Explanations
(u) A. Pulsus paradoxus is a classic finding for cardiac tamponade.
(u) B. Localized crackles are associated with pneumonia and consolidation, not pericarditis.
(c) C. Pericardial friction rub is characteristic of an inflammatory pericarditis.
(u) D. Wheezing is characteristic for pulmonary disorders, such as asthma.
Ref: (27)

5. History & Physical/Cardiology


A 65 year-old white female presents with dilated, tortuous veins on the medial aspect of
her lower extremities. Which of the following would be the most common initial
complaint?
Answers
A. Pain in the calf with ambulation
B. Dull, aching heaviness brought on by periods of standing
C. Brownish pigmentation above the ankle
D. Edema in the lower extremities
Explanations
(u) A. Patients with deep venous thrombosis (DVT) may present with complaints of pain in the
calf with ambulation. Secondary varicosities may result from DVT's.

(c) B. Dull, aching heaviness or a feeling of fatigue brought on by periods of standing is the most
common complaint of patients presenting initially with varicosities.
(u) C. Stasis Dermatitis and edema are most suggestive of chronic venous insufficiency.
(u) D. See C for explanation.
Ref: (27)

6. History & Physical/Cardiology


A 22 year-old male received a stab wound in the chest an hour ago. The diagnosis of
pericardial tamponade is strongly supported by the presence of
Answers
A. pulmonary edema.
B. wide pulse pressure.
C. distended neck veins.
D. an early diastolic murmur.
Explanations
(u) A. Pulmonary edema may result with low output states as seen with myocardial contusions,
but it is not strongly suggestive of tamponade.
(u) B. Wide pulse pressure is seen in conditions of high stroke volume such as aortic insufficiency
or hyperthyroidism. Narrow pulse pressure is seen with cardiac tamponade.
(c) C. Cardiac compression will manifest with distended neck veins and cold clammy skin.
(u) D. The onset of diastolic murmur is suggestive of valvular disease, not tamponade.
Ref: (28)

7. History & Physical/Dermatology


A patient presents with a rash, characterized by red macules and edematous papules with
a clearing center. This best describes which of the following?
Answers
A. erythema marginatum
B. erythema multiforme
C. varicella
D. impetigo
Explanations
(u) A. Erythema marginatum is associated with rheumatic fever and is characterized by macular
to maculopapular lesions. A clearing center is not found in the rash.
(c) B. Target lesions, also termed iris lesions, are characteristic of erythema multiforme. The rash
may be recurrent but typically resolves over 3-6 weeks.
(u) C. The rash of varicella typically has maculopapules, vesicles, and scabs in various stages of
development. A clearing center is not found in the rash.
(u) D. The lesions of impetigo are pustules that form a honey-colored crust after rupturing.

Ref: (8)

8. History & Physical/Dermatology


In a patient suspected of having seborrheic dermatitis, the most common site of
involvement would be the

10

Answers
A. upper extremities.
B. thighs.
C. scalp.
D. feet.
Explanations
(u) A. See C for explanation.
(u) B. See C for explanation.
(c) C. The most common site of involvement of seborrheic dermatitis is the scalp. Other common
sites include the eyebrows, eyelids, nasolabial fold, and ears.
(u) D. See C for explanation.
Ref: (8)

9. History & Physical/Endocrinology


A 26-year-old obese female complains of a 3-4 month history of discrete erythematous
plaques on the pretibial areas of her legs. The lesions have increased in size, become
darker, and are painful. She is concerned because the centers of the lesions have become
ulcerated. This patient should be screened for which of the following?
Answers
A. Hypothyroidism
B. Diabetes mellitus
C. Melanoma
D. Scleroderma
Explanations
(u) A. In hypothyroidism the skin of the pretibial area may thicken leading to edema. This is a
diffuse finding, involving the face and eyelids, without discrete lesions.
(c) B. The description of the skin lesions is characteristic of necrobiosis lipoidica diabeticorum,
one of the dermatologic manifestations of diabetes mellitus.
(u) C. The lesions of melanoma are typically not painful and do not ulcerate.
(u) D. Scleroderma is marked by thickening of the skin, with swelling of the fingers and hands.
The swelling may involve the forearms and face; the lower extremities are relatively spared.
Ref: (8)

10. History & Physical/Endocrinology


A 40 year-old male presents to your clinic complaining of nontender, yellow patches on
both eyelids. He states his brother and uncle have similar growths. He denies any visual
changes or other complaints. Your primary suspicion is
Answers
A. gout.
B. lipoma.
C. hyperlipidemia.
D. seborrheic dermatitis.
Explanations
(u) A. Tophaceous gout may appear as yellow skin lesions but they usually occur around the
joints and helix of the ear.
(u) B. Lipomas tend to be flesh-colored and are not usually bilateral.

11

(c) C. Xanthelasmas, along with xanthomas, are common findings in familial


hypercholesterolemia.
(u) D. Eyelids are a common location for seborrheic dermatitis but the lesions are not yellow in
color.
Ref: (8)

11. History & Physical/ENT/Ophthalmology


A 4 year-old child presents with a rapid onset of high fever and extremely sore throat.
Which of the following findings are suggestive of the diagnosis of epiglottitis?
Answers
A. Croupy cough and drooling
B. Thick gray, adherent exudate
C. Beefy red uvula, palatal petechiae, white exudate
D. Inflammation and medial protrusion of one tonsil
Explanations
(c) A. A croupy cough with drooling in a patient who appears very ill is consistent with epiglottitis.
Examining the throat is contraindicated, unless the airway can be maintained.
(u) B. Thick gray adherent exudate is suggestive of diphtheria.
(u) C. Beefy red uvula, palatal petechiae, and white exudate are findings suggestive of
streptococcal pharyngitis.
(u) D. Inflammation with medial protrusion of the tonsil is suggestive of a peritonsillar abscess.
Ref: (4)

12. History & Physical/ENT/Ophthalmology


Which of the following are normal findings in a Weber test?
Answers
A. The tympanic membrane is movable with pneumatic otoscopy.
B. The tympanic membrane is pearly gray with a sharp cone of light with apex at the umbo.
C. Sound is heard equally in both ears when a vibrating tuning fork is placed on the mid forehead.
D. Air conduction is greater than bone conduction when a vibrating tuning fork is moved from the
mastoid bone to close to the ear canal.
Explanations
(u) A. A movable tympanic membrane indicates there is no effusion, and is not the Weber test.
(u) B. The tympanic membrane is evaluated by direct observation with an otoscope, and is not
the Weber test.
(c) C. A normal Weber test means there is no lateralization of sound perception when a vibrating
tuning fork is placed on the mid forehead.
(u) D. A normal Rinne test means that tuning fork vibration is heard longer through the air than
the bone.
Ref: (4)

13. History & Physical/ENT/Ophthalmology


Which of the following is diagnosed by use of the cover/uncover test?
Answers
A. Adies pupil

12

B. Strabismus
C. Glaucoma
D. Myopia
Explanations
(u) A. Adies pupil is a sluggish pupil reaction to light and accommodation, evaluated by papillary
reaction to light.
(c) B. The cover/uncover test is used to diagnose strabismus.
(u) C. Tonometry is used to measure intraocular pressure to evaluate for glaucoma.
(u) D. Myopia is evaluated by using a Snellen chart.
Ref: (4)

14. History & Physical/Gastrointestinal/Nutritional


A patient is known to have end stage liver disease due to cirrhosis. Which of the following
physical examination findings would commonly be seen in this patient?
Answers
A. Testicular hypertrophy
B. Muscular pseudohypertrophy
C. Gynecomastia
D. Hepatomegaly
Explanations
(u) A. Testicular atrophy, wasting of the muscles of the lower extremity, spider angiomas, caput
medusa and gynecomastia are physical examination findings associated with end stage liver
disease associated due to cirrhosis.
(u) B. Muscular pseudohypertrophy is seen in muscular dystrophy.
(c) C. See A for explanation.
(u) D. Patients with end stage liver failure who have cirrhosis have a small shrunken liver from the
ongoing cellular destruction and fibrosis.
Ref: (8)

15. History & Physical/Gastrointestinal/Nutritional


Which of the following is the most consistent physical examination finding in a patient
with duodenal ulcer?
Answers
A. Flank tenderness
B. Right upper quadrant tenderness
C. Epigastric tenderness
D. Rebound tenderness
Explanations
(u) A. Flank tenderness is caused by urologic disorders such as pyelonephritis and renal lithiasis.
(u) B. Right upper quadrant tenderness on palpation is a typical feature for cholecystitis.
(c) C. Epigastric tenderness is a key feature of duodenal ulcer.
(u) D. Rebound tenderness is a feature of peritonitis from rupture of a hollow viscus and is not
seen with just the presence of duodenal ulcer.
Ref: (8)

13

16. History & Physical/Gastrointestinal/Nutritional


The initial sign or symptom of iron poisoning in a 3 year-old child is usually
Answers
A. vomiting and bloody diarrhea.
B. convulsions and tetany.
C. somnolence and coma.
D. ataxia and colicky abdominal pain.
Explanations
(c) A. Iron causes localized necrosis and hemorrhage at the point of contact in the GI system
resulting in abdominal pain, vomiting, bloody diarrhea, and hematemesis.
(u) B. Convulsions and tetany are symptoms of hypocalcemia.
(u) C. Somnolence and coma are not initial findings in iron ingestion.
(u) D. Ataxia and colicky abdominal pain are consistent with lead poisoning.
Ref: (28)

17. History & Physical/Gastrointestinal/Nutritional


A classic skin finding seen in patients with inflammatory bowel disease would be
Answers
A. erythematous plaques on the extremities.
B. poorly healing, indolent ulcers on the lower extremities.
C. pretibial myxedema.
D. purple striae.
Explanations
(u) A. Granuloma annulare is seen with diabetes mellitus. It consists of erythematous plaques on
the extremities or trunk.
(c) B. Pyoderma gangrenosum is classically seen with inflammatory bowel disease and is rarely
seen in the absence of inflammatory bowel disease.
(u) C. Pretibial myxedema is the skin manifestation of hyperthyroidism.
(u) D. The dermatologic manifestations of Cushing's disease are purple striae and a
supraclavicular fat pad.
Ref: (8)

18. History & Physical/Hematology


A 55-year-old non-smoking male presents with a hemoglobin of 18.5 g/dl and a hematocrit
of 56%. Which of the following physical examination findings is the most likely to be
noted with this patient?
Answers
A. Splenomegaly
B. Cheilosis
C. Purpura
D. Decreased vibratory sense
Explanations
(c) A. Patients with polycythemia vera present with elevated hemoglobin and hematocrit. On
physical examination plethora, engorged retinal veins, and splenomegaly are common.
(u) B. Cheilosis is noted in iron deficiency anemia.
(u) C. Purpura is typically noted in bleeding disorders.

14

(u) D. Decreased vibratory sense is noted in vitamin B12 deficiency.


Ref: (8)

19. History & Physical/Neurology


A 73 year-old male presents to the clinic with his wife. His wife has noticed that he has
developed a resting tremor in his right hand and a shuffling gait over the last year. What
finding on physical examination would support your suspected diagnosis?
Answers
A. Chorea
B. Dystonia
C. Masked facies
D. Hyperreflexia
Explanations
(u) A. See C for explanation.
(u) B. See C for explanation.
(c) C. The patient symptoms are consistent with Parkinsonism. Physical exam findings include
masked facies, micrographia, decreased arm swing, and monotonous speech.
(u) D. See C for explanation.
Ref: (27)

20. History & Physical/Neurology


A patient with an upper motor neuron lesion would exhibit which of the following
findings?
Answers
A. Fasciculations
B. Areflexia
C. Muscular atrophy
D. Spasticity
Explanations
(u) A. Fasciculations, areflexia and muscle atrophy are consistent with lower motor neuron
lesions.
(u) B. See A for explanations.
(u) C. See A for explanation.
(c) D. Spasticity is an upper motor neuron lesion finding.
Ref: (4)

21. History & Physical/Obstetrics/Gynecology


On examination of a pregnant patient the physician assistant notes a bluish or purplish
discoloration of the vagina and cervix. This is called
Answers
A. Hegar's sign.
B. McDonald's sign.
C. Cullen's sign
D. Chadwick's sign

15

Explanations
(u) A. Hegar's sign is the softening of the cervix that often occurs with pregnancy.
(u) B. McDonald's sign is when the uterus becomes flexible at the uterocervical junction at 7-8
weeks.
(u) C. Cullens sign is a purplish discoloration periumbilical and noted in pancreatitis.
(c) D. Chadwicks sign is a bluish or purplish discoloration of the vagina and cervix.
Ref: (9)

22. History & Physical/Obstetrics/Gynecology


On examination of a pregnant patient the physician assistant notes the fundal height is at
the level of the umbilicus. This corresponds to what gestational age?
Answers
A. 16 weeks
B. 20 weeks
C. 24 weeks
D. 28 weeks
Explanations
(u) A. See B for explanation.
(c) B. At 20-22 weeks the fundal height is typically at the level of the umbilicus.
(u) C. See B for explanation.
(u) D. See B for explanation.
Ref: (9)

23. History & Physical/Obstetrics/Gynecology


Which of the following is the most common manifestation of polycystic ovarian
syndrome?
Answers
A. Desquamation
B. Hirsutism
C. Galactorrhea
D. Rebound tenderness
Explanations
(u) A. Desquamation is noted in toxic shock syndrome.
(c) B. The patient with polycystic ovarian syndrome typically presents with hirsutism or infertility.
(u) C. Galactorrhea is noted in hyperprolactinemia.
(u) D. Rebound tenderness is noted in conditions causing peritonitis.
Ref: (9)

24. History & Physical/Orthopedics/Rheumatology


Abduction of the shoulder against resistance helps localize pain in which of the following
muscles of the shoulder girdle?
Answers
A. Supraspinatus
B. Infraspinatus
C. Teres minor

16

D. Subscapularis
Explanations
(c) A. Abduction against resistance tests the supraspinatus.
(u) B. Lateral rotation against resistance tests the infraspinatus and teres minor.
(u) C. See B for explanation.
(u) D. Medial rotation against resistance tests the subscapularis.
Ref: (4)

25. History & Physical/Orthopedics/Rheumatology


A 22 year-old male presents to the ED after sustaining a blow to the knee during football
practice. The knee exam demonstrates significant forward translation of the tibia when
the knee is in 15 degrees of flexion and external rotation at the hip. Which of the following
knee maneuvers does this represent?
Answers
A. Abduction stress test
B. Anterior drawer sign
C. Lachman test
D. McMurray test
Explanations
(u) A. The abduction stress test is performed to evaluate medial collateral ligament tears while
applying valgus stress.
(u) B. The anterior drawer sign is performed to evaluate the anterior cruciate ligament; however
the patient is supine, hips and knees flexed, and feet are flat on the table.
(c) C. The Lachman test is performed to evaluate the anterior cruciate ligament. The knee is
placed in 15 degrees of flexion and external rotation of the hip.
(u) D. The McMurray test is performed to evaluate medial and lateral meniscal tears while rotating
the lower leg internally and externally.
Ref: (4)

26. History & Physical/Orthopedics/Rheumatology


A 12 year-old female presents for a routine sports physical. The physical exam reveals
asymmetry of the posterior chest wall on forward bending. This is the most striking and
consistent abnormality of which of the following?
Answers
A. Spondylolysis
B. Spondolisthesis
C. Scoliosis
D. Herniated disc
Explanations
(u) A. Spondylolysis presents with limitation of lumbar flexibility and tight hamstring muscles.
(u) B. Spondylolisthesis presents with reduced lumbar lordosis and sacral kyphosis.
(c) C. Asymmetry of the posterior chest wall on forward bending is the most striking and
consistent abnormality in patients with idiopathic scoliosis.
(u) D. Herniated disc presents with lumbar muscle spasm and a positive straight leg test.
Ref: (6)

17

27. History & Physical/Orthopedics/Rheumatology


Physical exam findings in a 4 year-old child that include blue sclerae and recurrent
fractures indicates which of the following?
Answers
A. Ehlers-Danlos syndrome
B. Marfan syndrome
C. Achondroplasia
D. Osteogenesis imperfecta
Explanations
(u) A. Physical exam findings in Ehlers-Danlos include laxity and hypermobility of joints, mitral
valve prolapse, and associated degenerative arthritis.
(u) B. Children with Marfan syndrome have hypotonia, arachnodactyly, joint laxity and
dislocations.
(u) C. Children with achondroplasia are below normal standards on growth charts. They have
difficulty balancing their large heads when beginning to walk.
(c) D. Mild osteogenesis imperfecta presents with blue sclerae, history of recurrent fractures and
presenile deafness.
Ref: (6)

28. History & Physical/Psychiatry/Behavioral Medicine


Which of the following historical factors differentiates post-traumatic stress disorder from
acute stress disorder?
Answers
A. The inability of the person to recall an important aspect of the event.
B. Avoidance of stimuli that invokes recollections of the event.
C. A belief that their future has been foreshortened because of the event.
D. The presence of sleep disorder.
Explanations
(u) A. See C for explanation.
(u) B. See C for explanation.
(c) C. Post-traumatic stress disorder and acute stress disorder have many of the same
characteristics. A sense of a foreshortened future, such as not expecting a normal life span or a
career due to the trauma, distinguishes post-traumatic stress disorder from an acute stress
disorder. The other answers are common to both disorders.
(u) D. See C for explanation.
Ref: (15)

29. History & Physical/Psychiatry/Behavioral Medicine


A patient with obsessive-compulsive disorder would most likely have which of the
following findings?
Answers
A. Raw, red hands
B. Priapism
C. Memory impairment
D. Abdominal pain
Explanations

18

(c) A. Common manifestations of obsessive-compulsive disorder include phobias of germ and


contaminants, which results in frequent hand washing leading to chafe and reddened hands. The
other answers are inconsistent with obsessive-compulsive disorder.
(u) B. See A for explanation.
(u) C. See A for explanation.
(u) D. See A for explanation.
Ref: (8)

30. History & Physical/Pulmonology


A 45 year-old male presents with sudden onset of pleuritic chest pain, productive cough
and fever for 1 day. He relates having symptoms of a cold for the past week that
suddenly became worse yesterday. Which of the following findings will most likely be
seen on physical examination of this patient?
Answers
A. spoken ee heard as ay
B. hyperresonant percussion note
C. wheezes over the involved area
D. vesicular breath sounds over involved area
Explanations
(c) A. This patient most likely has a bacterial pneumonia with consolidation, which would produce
egophony, where a spoken ee is heard as ay.
(u) B. Consolidation from bacterial pneumonia causes findings of dullness to percussion, late
inspiratory crackles and bronchial breath sounds over the involved area.
(u) C. See explanation B.
(u) D. See explanation B.
Ref: (4)

31. History & Physical/Pulmonology


Which of the following is a common symptom associated with laryngotracheobronchitis
(viral croup)?
Answers
A. drooling
B. high fever
C. "hot potato" voice
D. barking cough
Explanations
(u) A. Drooling and a "hot potato" voice are seen with epiglottitis, not viral croup.
(u) B. Fever is usually absent or low grade in patients with viral croup.
(u) C. See A for explanation.
(c) D. Viral croup is characterized by history of an upper respiratory tract symptoms followed by
onset of a barking cough and stridor.
Ref: (14)

32. History & Physical/Pulmonology


A foreign body lodged in the trachea that is causing partial obstruction will most likely
produce what physical examination finding?

19

Answers
A. stridor
B. aphonia
C. inability to cough
D. progressive cyanosis
Explanations
(c) A. An inspiratory wheeze is called stridor, which indicates a partial obstruction of the trachea
or larynx.
(u) B. Aphonia, inability to cough and progressive cyanosis are seen with complete obstruction of
the trachea, not partial obstruction.
(u) C. See B for explanation.
(u) D. See B for explanation.
Ref: (14)

33. History & Physical/Pulmonology


On physical examination you note diminished breath sounds over the right lower lobe with
decreased tactile fremitus and dullness to percussion. Which of the following is the most
likely cause?
Answers
A. asthma
B. consolidation
C. pneumothorax
D. pleural effusion
Explanations
(u) A. Asthma is characterized by decreased tactile fremitus, but would have resonant to
hyperresonant percussion, not dullness.
(u) B. Consolidation from pneumonia is characterized by dullness to percussion, but would have
an increased, not decreased, tactile fremitus.
(u) C. A pneumothorax is characterized by decreased to absent tactile fremitus, but would have a
hyperresonant percussion note, not dullness.
(c) D. A decreased tactile fremitus and dullness to percussion would be found in a pleural
effusion.
Ref: (4)

34. History & Physical/Urology/Renal


A patient with a 15-year history of type 2 diabetic mellitus presents for follow-up. Labs
reveal a BUN 100 mg/dl, serum creatinine 9.2 mg/dl, and serum glucose 164 mg/dl. Which
of the following would you expect to find on physical examination?
Answers
A. Pruritus
B. Hypotension
C. Macroglossia
D. Suprapubic tenderness
Explanations
(c) A. Hypertension, pruritus and xerosis are common findings in the uremic patient.
(u) B. See A for explanation.

20

(u) C. See A for explanation.


(u) D. Suprapubic tenderness is associated with urinary tract infection or acute obstructive
uropathy.
Ref: (27)

35. History & Physical/Urology/Renal


When performing a rectal examination, prostatic massage is contraindicated in
Answers
A. prostatodynia.
B. non-bacterial prostatitis.
C. chronic bacterial prostatitis.
D. acute bacterial prostatitis.
Explanations
(u) A. Prostatodynia is an inflammatory disorder involving voiding dysfunction and pelvic floor
musculature dysfunction. There is no bacterial involvement.
(u) B. Non-bacterial prostatitis is similar to chronic bacterial prostatitis, but no bacteria are
cultured, and the cause is unknown.
(u) C. Prostate massage can be performed in the absence of fever. Expressed prostatic
secretions are cultured to help identify the organism.
(c) D. Vigorous manipulation of the prostate during rectal examination may result in septicemia.
This is contraindicated in the presence of fever, irritative voiding symptoms, and perineal/sacral
pain.
Ref: (12)

36. History & Physical/Infectious Diseases


Which of the following is typically noted on physical examination in a patient with
diphtheria?
Answers
A. Papular rash on trunk
B. Supraclavicular adenopathy
C. Pharyngeal pseudomembranes
D. Splenomegaly
Explanations
(u) A. See C for explanation.
(u) B. See C for explanation.
(c) C. The classic exam finding noted in diphtheria is a gray pharyngeal pseudomembrane.
Rash, splenomegaly, and supraclavicular adenopathy are not noted in diphtheria.
(u) D. See C for explanation.
Ref: (8)

37. Diagnostic Studies/Cardiology


Cardiac nuclear scanning is done to detect
Answers
A. electrical conduction abnormalities.
B. valvular abnormalities.

21

C. ventricular wall dysfunction.


D. coronary artery patency/occlusion.
Explanations
(u) A. An EKG is used to determine electrical conduction abnormalities.
(u) B. An echocardiogram is a non-invasive test used to determine valvular abnormalities and wall
motion.
(c) C. Visualization of the cardiac wall can be done with cardiac nuclear scanning. This is done to
determine hypokinetic areas from akinetic areas.
(u) D. Patency or occlusion is assessed with cardiac catheterization (invasive).
Ref: (12)

38. Diagnostic Studies/Cardiology


A 72 year-old male with a new diagnosis of congestive heart failure and atrial fibrillation,
develops episodes of hemodynamic compromise secondary to increased ventricular rate.
A decision to perform elective cardioversion is made and the patient is anticoagulated
with heparin. Which test should be ordered to assess for atrial or ventricular mural
thrombi?
Answers
A. Electrocardiogram
B. Chest x-ray
C. Transesophageal Echocardiogram
D. C-reactive protein
Explanations
(u) A. Electrical conduction will not assess for mural thrombi.
(u) B. A chest x-ray will not visualize the left atria and ventricles to assess for mural thrombi.
(c) C. Transesophageal echocardiography allows for determination of mural thrombi that may
have resulted from atrial fibrillation.
(u) D. C-reactive protein is not going to give you any information regarding thrombi. This test is
used to identify the presence of inflammation.
Ref: (12)

39. Diagnostic Studies/Cardiology


A 64 year-old patient with known history of type 1 diabetes mellitus for 50 years has
developed pain radiating from the right buttock to the calf. Patient states that the pain is
made worse with walking and climbing stairs. Based upon this history which of the
following would be the most appropriate test to order?
Answers
A. Venogram
B. Arterial duplex scanning
C. X-ray of the right hip and L/S spine
D. Venous Doppler ultrasound
Explanations
(u) A. See B for explanation.
(c) B. Given the patient's long history of type 1 diabetes mellitus the patient most likely has
vascular occlusive disease. Evaluation of arterial blood flow is assessed using the duplex

22

scanner. X-ray of the L/S spine and right hip while not harmful may give information regarding
bony structures. Venous Doppler ultrasound will not give information of arterial perfusion.
(u) C. See B for explanation.
(u) D. See B for explanation.
Ref: (12)

40. Diagnostic Studies/Cardiology


A 36 year-old male complains of occasional episodes of "heart fluttering". The patient
describes these episodes as frequent, short-lived and episodic. He denies any associated
chest pain. Based on this information, which one of the following tests would be the most
appropriate to order?
Answers
A. Holter monitor
B. Cardiac catheterization
C. Stress testing
D. Cardiac nuclear scanning
Explanations
(c) A. Holter monitoring is a non-invasive test done to obtain a continuous monitoring of the
electrical activity of the heart. This can help to detect cardiac rhythm disturbances that can
correlate with the patient symptoms. Cardiac catheterization is an invasive procedure done to
assess coronary artery disease. Stress testing and cardiac nuclear scanning are non-invasive
testing maneuvers done to assess coronary artery disease.
(u) B. See A for explanation.
(u) C. See A for explanation.
(u) D. See A for explanation.
Ref: (12)

41. Diagnostic Studies/Cardiology


A patient with a mitral valve replacement was placed post-operatively on warfarin
(Coumadin) for anticoagulation prophylaxis. To monitor this drug for its effectiveness,
what test would be used?
Answers
A. PTT
B. PT-INR
C. Platelet aggregation
D. Bleeding time
Explanations
(u) A. PTT is a reflection of the intrinsic clotting system and is used to monitor heparin
administration.
(c) B. PT-INR is a reflection of the extrinsic and common pathway clotting system. Coumadin
interferes with Vitamin K synthesis which is needed in the manufacture of factors II, VII, IX, X
which are part of the extrinsic clotting pathway.
(u) C. Platelet aggregation tests are utilized to assess platelet dysfunction.
(u) D. Bleeding time is used to assess platelet function.
Ref: (12)

23

42. Diagnostic Studies/Dermatology


A 26 year-old male complains of intense itching, especially at night and after hot showers,
for the past 4 days. On physical examination he has a few red papules and areas of
excoriation on his volar wrists, between his fingers, and around his waist. Proper
diagnosis should include which of the following tests?
Answers
A. KOH prep
B. Gram stain
C. Skin scraping microscopy
D. Tzanck prep
Explanations
(u) A. A KOH prep would be used to examine for evidence of a fungal infection.
(u) B. A Gram stain would be used for a bacterial infection and would be inappropriate in this
situation.
(c) C. The history and exam is consistent with a scabies infection. Scrapings from the burrows
should be examined for the presence of mites, eggs, and feces.
(u) D. A Tzanck prep would be used to examine for giant multinucleated cells characteristic of a
herpes infection.
Ref: (8)

43. Diagnostic Studies/Dermatology


A 35 year-old female who recently returned from a backpacking trip complains of fatigue,
malaise, fever, chills, and arthralgias. Physical examination reveals a 6 cm annular lesion
with a red border and a clear center on her mid-back. Which of the following laboratory
tests would support your diagnosis?
Answers
A. KOH prep of skin scrapings
B. Blood cultures
C. RAST testing
D. Serologic antibody testing
Explanations
(u) A. Although the skin lesion may resemble a fungal infection, a fungal dermatophyte would not
present with systemic symptoms.
(u) B. Culturing of <i>Borrelia burgdorferi</i> from clinical specimens, with the exception of skin
biopsies at the site of the lesion, have resulted in low yields.
(u) C. RAST testing is utilized in evaluation of allergies and is not indicated in this situation.
(c) D. Most people with Lyme Disease will have a positive serologic test after the first few weeks
of infection and this would support the diagnosis.
Ref: (8)

44. Diagnostic Studies/Endocrinology


A patient complains of fatigue, tremors, palpitations, and heat intolerance. The thyroid is
diffusely enlarged and firm on palpation. Which of the following laboratory findings is the
most consistent with this presentation?
Answers
A. Low T4

24

B. Low TSH
C. Decreased bilirubin
D. Normal radionuclide scan
Explanations
(u) A. See B for explanation.
(c) B. The presentation is consistent with hyperthyroidism. Laboratory findings include low TSH,
elevated free and total thyroid hormone levels, and an increased uptake on radionuclide scan.
There may also be elevated bilirubin, liver enzymes, and ferritin levels, along with anemia and
thrombocytopenia.
(u) C. See B for explanation
(u) D. See B for explanation
Ref: (8)

45. Diagnostic Studies/Endocrinology


A solitary thyroid nodule is noted on physical examination. The TSH level is normal. The
next step in the evaluation is
Answers
A. measurement of T4 and free T3 levels.
B. a radionuclide thyroid scan.
C. a fine needle biopsy.
D. a surgical excision.
Explanations
(u) A. Measurement of T4 and T3 levels would not be of benefit in the evaluation of a solitary
thyroid nodule with a normal TSH level.
(u) B. A thyroid scan would be the next step if there were a low TSH level.
(c) C. Fine needle aspiration (FNA) is the first step in the evaluation of a solitary nodule with a
normal TSH level. FNA has a high level of accuracy in diagnosing benign versus malignant
nodules in this setting.
(u) D. Surgical excision would be the final step after determination of malignancy or suspicion of
malignancy by FNA.
Ref: (8)

46. Diagnostic Studies/ENT/Ophthalmology


A 32 year-old carpenter complains of right eye irritation all day after driving a metal stake
into the ground with his hammer. He states that something flew into my eye. Visual
acuity is 20/20. Pupils are equal, round, reactive to light and accommodation. Extraocular
movements are intact. There is minimal right corneal injection. No foreign body is noted
with lid eversion. Fluorescein stain reveals a tiny pinpoint uptake in the area of the
corneal injection. Which of the following is the most appropriate diagnostic test at this
stage?
Answers
A. MRI
B. X-ray orbits
C. Applanation tonometry
D. Fluorescein angiography
Explanations

25

(h) A. MRI should never be used when there is suspicion of an iron-containing intraocular foreign
body.
(c) B. Orbital x-rays or CT scan will be most helpful in identifying an intraocular metallic foreign
body.
(u) C. Tonometry is used to evaluate intraocular pressure, but not the presence of intraocular
foreign bodies.
(u) D. Fluorescein angiography is used to evaluate vessels of the eye, not intraocular foreign
bodies.
Ref: (30)

47. Diagnostic Studies/ENT/Ophthalmology


A 45 year-old male complains of loss of hearing in his left ear. He also complains of
ringing in the ear, and has had occasional dizziness. On exam, there is unilateral leftsided sensorineural hearing loss and a diminished corneal reflex. Neuro exam is
otherwise normal. TMs are normal, and canals are clear. Neck is supple, without
adenopathy. Oropharynx is normal. Of the following, the best diagnostic study to identify
the cause of this patients complaints is
Answers
A. auditory brainstem evoked response.
B. gadolinium-enhanced MRI.
C. acoustic reflex testing.
D. vestibular testing.
Explanations
(u) A. See B for explanation.
(c) B. MRI has replaced auditory brainstem evoked response and acoustic reflex testing in the
evaluation of patients for acoustic neuromas.
(u) C. See B for explanation.
(u) D. Vestibular testing is not a useful screening test for acoustic neuromas.
Ref: (12)

48. Diagnostic Studies/Gastrointestinal/Nutritional


Which of the following laboratory abnormalities is most commonly seen acutely in a
patient who has a massive GI bleed?
Answers
A. Increased BUN level
B. Hypercalcemia
C. Hyponatremia
D. Increased AST
Explanations
(c) A. Blood in the gut will cause a considerable increase in the BUN that is independent of
decreased renal perfusion or intrinsic renal dysfunction. BUN rises as a result of catabolism and
absorption of blood protein with a resultant increase in nitrogenous waste.
(u) B. Acute blood loss does not result in a change in the calcium level unless multiple
transfusions are given.
(u) C. Massive GI blood loss acutely results in blood volume contraction without acutely changing
the sodium concentration until intravenous therapy is given
(u) D. Increases in the serum AST is the result of hepatocyte injury or inflammation and does not
occur as a result of GI bleeding.

26

Ref: (8)

49. Diagnostic Studies/Gastrointestinal/Nutritional


Primary biliary cirrhosis will have which of the following laboratory results?
Answers
A. Decreased haptoglobin
B. Anticholinesterase antibodies
C. Antimitochondrial antibodies
D. Elevated ceruloplasmin
Explanations
(u) A. Haptoglobin is a glycoprotein that is made in the liver that acts as a scavenger molecule to
recapture iron after hemolysis occurs. Its levels decrease with active bleeding or cell destruction
as seen in hemolytic anemia.
(u) B. Anticholinesterase antibodies are evaluated in patients suspected of having myasthenia
gravis.
(c) C. Antimitochondrial antibodies are seen in patients with primary biliary cirrhosis, a chronic,
progressive cholestatic disease of the liver that is characterized by destruction of the extrahepatic
bile ducts.
(u) D. Ceruloplasmin elevations are seen with Wilsons disease, a disease that is due to a
disordered copper metabolism.
Ref: (8)

50. Diagnostic Studies/Gastrointestinal/Nutritional


Which of the following diagnostic tests is considered to be the best initial test to order in a
patient with suspected gallbladder disease?
Answers
A. Ultrasound
B. Hepatic iminodiacetic acid (HIDA) scan
C. Flat plate of the abdomen
D. Endoscopic retrograde cholangiopancreatography (ERCP)
Explanations
(c) A. Ultrasound of the abdomen is the best test for checking the extra-hepatic biliary tree for
ductal dilatation and choledocholithiasis.
(u) B. HIDA scan is usually ordered to assess gallbladder function. It is mostly ordered if initial
ultrasound is normal and there is still a high index of suspicion for gallbladder disease.
(u) C. Flat plate of the abdomen will only identify about 10 to 15% of gallstones.
(u) D. ERCP is performed to remove gallstones that have become lodged in the common bile
duct. It is not an initial study that is performed.
Ref: (8)

51. Diagnostic Studies/Hematology


An 8-year-old presents with splenomegaly. CBC results reveal the following: WBC6,300/microliter, Hgb- 10.5 g/dl, Hct- 31%, MCV- 87 fL, MCHC- 39 g/dl, MCH- 28 pg, and
platelets- 317,000/mL. Examination of the RBC morphology reveals 80% spherocytes.
Which of the following would be most helpful in confirming the diagnosis?

27

Answers
A. Direct Coombs test
B. Osmotic fragility
C. G-6-PD level
D. Serum ferritin
Explanations
(u) A. The direct Coombs test would be negative and would not be helpful in diagnosing
hereditary spherocytosis.
(c) B. Hereditary spherocytosis presents with a normocytic, normochromic anemia and many
spherocytes. Diagnosis is confirmed with a positive osmotic fragility test.
(u) C. G-6-PD deficiency presents with minimal or no RBC morphologic abnormalities and is
diagnosed by measuring G-6-PD enzyme activity level.
(u) D. Iron deficiency anemia typically presents with microcytic, hypochromic red blood cells and
is diagnosed with a serum ferritin.
Ref: (8)

52. Diagnostic Studies/Neurology


What test is the single most useful test in establishing the diagnosis of multiple sclerosis?
Answers
A. Cerebral spinal fluid cell count and protein level
B. Cerebral spinal fluid immunoglobulin studies
C. Evoked potentials
D. Magnetic Resonance Imaging
Explanations
(u) A. While cerebral spinal fluid cell count, protein levels, and immunoglobins may be abnormal
they are not specific for multiple sclerosis.
(u) B. See A for explanation.
(u) C. Evoked potentials are most useful in the detection of subclinical involvement of
neuropathways in MS, but does not establish the diagnosis.
(c) D. The presence of plaques on MRI is a key finding in establishing the diagnosis of MS.
Ref: (27)

53. Diagnostic Studies/Neurology


A 22 year-old male presents to the clinic complaining of excessive daytime somnolence
and strong desires to sleep at inappropriate times. He came in today because he had an
episode of "feeling paralyzed" as he was falling asleep yesterday. What is the most
appropriate diagnostic test to confirm this patients diagnosis?
Answers
A. MRI of the brain
B. Electroencephalogram
C. Multiple sleep latency test
D. Overnight polysomnography
Explanations
(u) A. See C for explanation.
(u) B. See C for explanation.
(c) C. Multiple sleep latency test is required to observe the abrupt transition to REM sleep and
establish the diagnosis of narcolepsy.

28

(u) D. See C for explanation.


Ref: (27)

54. Diagnostic Studies/Obstetrics/Gynecology


A 25 year-old female presents with vulvar pruritus and a thick, white vaginal discharge.
Which of the following tests will be most helpful in making the correct diagnosis?
Answers
A. KOH prep
B. Gram stain
C. Tzanck smear
D. FTA-ABS
Explanations
(c) A. KOH prep is used to assist in the diagnosis of vaginal candidiasis, which presents with
vulvar pruritus and white curd like, cheesy vaginal discharge.
(u) B. Gram stain is used in the diagnosis of bacterial infections.
(u) C. Tzanck smear is used to diagnose herpes infections.
(u) D. FTA-ABS is used to diagnose syphilis.
Ref: (9)

55. Diagnostic Studies/Obstetrics/Gynecology


A 25 year-old presents with pelvic pain and uterine bleeding. Her Beta-HCG was 1200
mIU/L six days ago. Her current Beta-HCG is 1600 mIU/L. What is the next best test in the
evaluation of this patient?
Answers
A. Laparoscopy
B. Culdocentesis
C. Dilation and curettage
D. Transvaginal ultrasound
Explanations
(u) A. The use of laparoscopy in the diagnosis of an ectopic pregnancy has decreased, but is still
useful when a definitive diagnosis is difficult.
(u) B. Culdocentesis is used in the diagnosis of intraperitoneal bleeding, which may or may not be
present in an ectopic pregnancy.
(u) C. Dilation and curettage may confirm or exclude intrauterine pregnancy but is not the next
best test in the evaluation of ectopic pregnancy.
(c) D. Transvaginal ultrasound is the best test to separate ectopic from intrauterine pregnancy.
Ref: (9)

56. Diagnostic Studies/Obstetrics/Gynecology


A couple presents having not been able to conceive over the past 12 months. Evaluation
of the male has been normal. The female has had regular menses. Ovulation can be
confirmed with mid-luteal phase measurement of which of the following?
Answers
A. Thyroid stimulating hormone
B. Luteinizing hormone

29

C. Progesterone
D. Prolactin
Explanations
(u) A. TSH is used only if signs of thyroid disease are present.
(u) B. LH, FSH, and prolactin are used to confirm ovulation in patients with irregular menstrual
cycles.
(c) C. Ovulation can best be confirmed by measuring serum progesterone levels in the mid-luteal
phase.
(u) D. See B for explanation.
Ref: (9)

57. Diagnostic Studies/Orthopedics/Rheumatology


A 65 year-old female presents to the office with a six-month history of back pain. The
patient states that she is shrinking and thinks she is about an inch shorter than she was a
year ago. Serum parathyroid hormone, calcium, phosphorus, and alkaline phosphatase
are all normal. Which of the following would you most likely see on the x-ray of her spine?
Answers
A. Radiolucent lesions
B. Demineralization
C. Chondrocalcinosis
D. Subperiosteal resorption
Explanations
(u) A. Pagets disease of bone presents with bone pain, kyphosis, bowed tibias, large head, and
deafness. The initial lesions are destructive and radiolucent. Pagets disease has a normal
serum calcium and phosphate, but the serum alkaline phosphatase is elevated.
(c) B. Osteoporosis presents with varying degrees of back pain and loss of height is common.
The serum calcium, parathyroid hormone, phosphorus, and alkaline phosphatase are normal. Xray findings demonstrate demineralization in the spine and pelvis.
(u) C. Chondrocalcinosis is the presence of calcium-containing salts in articular cartilage and is
commonly seen in hyperparathyroidism, diabetes, hypothyroidism, and gout.
(u) D. Hyperparathyroidism is frequently asymptomatic. Serum parathyroid hormone and serum
calcium are elevated. X-ray findings include demineralization, subperiosteal resorption of bone
especially in the radial aspects of the fingers.
Ref: (27)

58. Diagnostic Studies/Orthopedics/Rheumatology


In a trauma patient who has a suspected cervical spine injury, the x-ray view that will
identify the majority of significant injuries is
Answers
A. lateral.
B. oblique.
C. anteroposterior.
D. odontoid.
Explanations
(c) A. The lateral view shows 70-80% of significant injuries. It is important to visualize all seven
cervical vertebrae and the upper margin of T1 to avoid missing possible pathology.

30

(u) B. The oblique view is usually not included in the initial set of x-rays taken. Bilateral supine
oblique is a view that may be ordered if all seven cervical vertebrae are not seen on the lateral
view.
(u) C. Anteroposterior view shows < 1% of significant injuries.
(u) D. The odontoid view reveals 10% of significant injuries.
Ref: (28)

59. Diagnostic Studies/Orthopedics/Rheumatology


A 38 year-old male sustained a fracture of the left distal tibia following a 25-foot fall and is
taken to the operating room for an open reduction internal fixation of the distal tibia.
Sixteen hours post-op, the patient develops sustained pain, which is not relieved with
narcotics. On passive range of motion of the toes the patient "yells" in agony. The patient
also states that the top of his foot has decreased sensation. On physical examination the
physician assistant notes that the leg is swollen and the foot is cool to touch. Based upon
this information what diagnostic testing should be done?
Answers
A. X-ray of the lower leg and ankle.
B. Doppler studies.
C. Bone scan.
D. Compartment pressure
Explanations
(u) A. X-rays of the lower leg and ankle will only determine bone placement.
(u) B. Doppler studies will confirm the presence of a decreased pulse.
(u) C. A bone scan is not indicated in the evaluation of compartment syndrome.
(c) D. Compartmental pressures should be obtained as soon as possible. If they are elevated this
is a surgical emergency.
Ref: (31)

60. Diagnostic Studies/Psychiatry/Behavioral Medicine


A 19 year-old female presents with complaints of intermittent abdominal pain associated
with recent, frequent episodes of regurgitation of food for the past several months and
worsening over the past 12 hours. She maintains a normal weight for her height however
she seems obsessed with losing weight. On examination the physician assistant notes
multiple dental caries, bilateral tenderness of the parotid glands and mild epigastric
tenderness. Which of the following findings would you expect to find on laboratory tests
to support your suspected diagnosis?
Answers
A. hypokalemia
B. hypocalcemia
C. hyperchloremia
D. hypermagnesemia
Explanations
(c) A. This patient most likely has bulimia nervosa - purging type. Self-induced vomiting is the
most common method of purging and this is supported by the physical examination findings noted
in this patient. Laboratory findings to support this diagnosis include hypochloremia with
subsequent hypokalemia due to renal compensatory mechanisms, hypomagnesemia and
metabolic alkalosis.
(u) B. See A for explanation.

31

(u) C. See A for explanation.


(u) D. See A for explanation.
Ref: (14)

61. Diagnostic Studies/Psychiatry/Behavioral Medicine


A divorced female patient presents for an employment physical. She states she has had a
"run of bad luck" with jobs and has not been able to hold any job for longer than 2-3
months. She also states she has been arrested several times for getting into fights when
she is out with the girls. She states she drinks an occasional beer, but denies any
significant problems with alcohol. Which of the following laboratory findings would
support your suspected diagnosis?
Answers
A. decreased triglycerides
B. decreased serum uric acid
C. increased LDL cholesterol
D. increased mean corpuscular volume
Explanations
(u) A. See D for explanation.
(u) B. See D for explanation.
(u) C. The primary lipid abnormalities demonstrated with alcoholism are increased triglycerides
and increased HDL cholesterol, not LDL cholesterol.
(c) D. This patient most likely has alcohol abuse as evidence by her social, occupational and legal
issues. Laboratory tests will reveal the presence of an elevated mean corpuscular volume,
triglycerides, serum uric acid and liver function tests.
Ref: (27)

62. Diagnostic Studies/Pulmonology


Which of the following is essential to make a diagnosis of cystic fibrosis?
Answers
A. Positive family history
B. Elevated sweat chloride
C. Recurrent respiratory infections
D. Elevated trypsinogen levels
Explanations
(u) A. Cystic fibrosis is a genetic disease, but a positive family history in and of itself is not
enough to diagnose the condition.
(c) B. The diagnosis of cystic fibrosis is made only after an elevated sweat chloride test or
demonstration of a genotype consistent with cystic fibrosis.
(u) C. While recurrent respiratory infections are a classic presentation of cystic fibrosis, the
diagnosis relies on confirmation, as noted in explanation B.
(u) D. Trypsinogen levels are used as a neonatal screening test and if elevated should be
followed by more definitive testing to confirm the diagnosis.
Ref: (6)

32

63. Diagnostic Studies/Pulmonology


An adult patient who is HIV positive receives a PPD. He develops an area of induration
that measures 8 mm after 48 hours. Which of the following is the most appropriate
interpretation of this test result?
Answers
A. positive
B. negative
C. active infection
D. falsely negative
Explanations
(c) A. A reaction size of greater than or equal to 5 mm in a HIV positive patient is considered a
positive tuberculin skin test reaction.
(u) B. See A for explanation.
(u) C. A positive PPD identifies patients that have been infected with Mycobacterium tuberculosis,
but does not indicate whether the disease is currently active or inactive.
(u) D. See A for explanation.
Ref: (27)

64. Diagnostic Studies/Pulmonology


A 23 year-old female with history of asthma for the past 5 years presents with complaints
of increasing shortness of breath for 2 days. Her asthma has been well controlled until 2
days ago and since yesterday she has been using her albuterol inhaler every 4-6 hours.
She is normally very active, however yesterday she did not complete her 30 minutes
exercise routine due to increasing dyspnea. She denies any cough, fever, recent
surgeries or use of oral contraceptives. On examination, you note the presence of
prolonged expiration and diffuse wheezing. The remainder of the exam is unremarkable.
Which of the following is the most appropriate initial diagnostic evaluation prior to
initiation of treatment?
Answers
A. chest x-ray
B. sputum gram stain
C. peak flow
D. ventilation perfusion scan
Explanations
(u) A. A chest x-ray should be ordered in an asthmatic patient only if you are concerned about the
presence of pneumonia or pneumothorax, neither of which is supported by the H&P findings
noted above.
(u) B. A sputum gram stain is performed in patients who you suspect have an infectious process,
such as pneumonia.
(c) C. A peak flow reading will help you to gauge her current extent of airflow obstruction and is
helpful in monitoring the effectiveness of any treatment interventions.
(u) D. A ventilation-perfusion scan (V/Q scan) is indicated in cases of suspected pulmonary
embolism. The patient above does not have any risk factors that would lead you to suspect such
a diagnosis.
Ref: (27)

33

65. Diagnostic Studies/Pulmonology


A patient presents with a history of progressive worsening of dyspnea over the past
several years. He gives a history of having worked as a ship builder for over 50 years. He
denies any alcohol or tobacco use. On examination you note clubbing and inspiratory
crackles. Which of the following chest x-ray findings support your suspected diagnosis?
Answers
A. hyperinflation and flat diaphragms
B. interstitial fibrosis and pleural thickening
C. cavitary lesions involving the upper lobes
D. eggshell calcification of hilar lymph nodes
Explanations
(u) A. Chest x-ray findings of hyperinflation and flat diaphragms suggest long-standing chronic
obstructive lung disease.
(c) B. This patient most likely has asbestosis, which is supported by his occupation as a ship
builder and clinical presentation as noted above. Chest x-ray findings include interstitial fibrosis,
pleural thickening and calcified pleural plaques on the diaphragm or lateral chest wall.
(u) C. Chest x-ray findings of cavitary lesions involving the upper lobes suggest pulmonary
tuberculosis.
(u) D. Chest x-ray findings of eggshell calcification of hilar lymph nodes strongly supports the
diagnosis of silicosis.
Ref: (27)

66. Diagnostic Studies/Urology/Renal


A 38 year-old female presents with right flank pain for several days, shaking chills, fever to
102F, and general malaise. The flank pain has been intermittently severe, and she has a
history of kidney stones. Urinalysis reveals 3+ red blood cells, 3+ leukocyte esterase, trace
protein and negative glucose. Which of the following findings would most likely be seen
on a renal ultrasound?
Answers
A. Small echogenic kidneys
B. Cysts
C. Hydronephrosis
D. Capsular hemorrhage
Explanations
(u) A. Small echogenic kidneys bilaterally, less than 10cm, support a diagnosis of chronic renal
failure.
(u) B. Cysts and capsular hemorrhage are not causes of obstructive pyelonephritis.
(c) C. Hydronephrosis, dilation of the collecting ducts, may be present due to a stone or other
source of obstruction.
(u) D. See B for explanation.
Ref: (8)

67. Diagnostic Studies/Urology/Renal


A 65 year-old patient presents with hypertension and peripheral edema. Urinalysis reveals
pale urine, with a specific gravity of 1.002, 2+ protein, trace glucose, and is negative for
red blood cells and leukocytes. Serum electrolytes include BUN of 58 mg/dl and creatinine
of 4.5 mg/dl. These are unchanged from previous results obtained 3 months and 6 months
ago. Of the following, what other laboratory abnormalities would you expect?

34

Answers
A. Hypercalcemia
B. Metabolic alkalosis
C. Hypophosphatemia
D. Anemia
Explanations
(u) A. Patients with chronic renal failure typically present with hypocalcemia, hyperphosphatemia,
and metabolic acidosis.
(u) B. See A for explanation.
(u) C. See A for explanation.
(c) D. Anemia of chronic disease is associated with chronic renal failure.
Ref: (8)

68. Diagnostic Studies/Infectious Diseases


An 8 year-old patient presents with fever, nausea, vomiting, and diarrhea, 12 hours after
playing with a turtle. The stools are watery, non-bloody and of moderate volumes. Which
of the following laboratory tests will be most helpful in making the diagnosis?
Answers
A. Stool for ova and parasites
B. Scotch tape test
C. Widal test
D. Stool culture
Explanations
(u) A. The most likely diagnosis is salmonellosis, which is a bacterial infection, and this will not be
diagnosed with ova and parasite studies.
(u) B. The scotch tape test is used to diagnose pinworm infections, which typically present with
perianal itching.
(u) C. The Widal test detects febrile agglutinins seen in typhoid fever, but a large number of falsepositives and false-negatives make this test not useful clinically.
(c) D. Salmonellosis presents with fever, nausea, vomiting and diarrhea, 6-48 hours after
ingestion of the organism. It is commonly transmitted to humans from eggs, poultry, and reptiles.
Diagnosis is made by isolation of the organism via stool culture.
Ref: (8)

69. Diagnosis/Cardiology
A 64 year-old male, with a long history of COPD, presents with increasing fatigue over the
last three months. The patient has stopped playing golf and also complains of decreased
appetite, chronic cough and a bloated feeling. Physical examination reveals distant heart
sounds, questionable gallop, lungs with decreased breath sounds at lung bases and the
abdomen reveals RUQ tenderness with the liver two finger-breadths below the costal
margin, the extremities show 2+/4+ pitting edema. Labs reveal the serum creatinine level
1.6 mg/dl, BUN 42 mg/dl, liver function test's mildly elevated and the CBC to be normal.
Which of the following is the most likely diagnosis?
Answers
A. Right ventricular failure
B. Pericarditis
C. Exacerbation of COPD

35

D. Cirrhosis
Explanations
(c) A. Signs of right ventricular failure are fluid retention i.e. edema, hepatic congestion and
possibly ascites.
(u) B. See A for explanation.
(u) C. See A for explanation.
(u) D. See A for explanation.
Ref: (8)

70. Diagnosis/Cardiology
A 56 year-old male with a known history of polycythemia suddenly complains of pain and
paresthesia in the left leg. Physical examination reveals the left leg is cool to the touch
and the toes are cyanotic. The popliteal pulse is absent by palpation and Doppler. The
femoral pulse is absent by palpation but weak with Doppler. The right leg and upper
extremities has 2+/4+ pulses throughout. Given these findings what is the most likely
diagnosis?
Answers
A. Venous thrombosis
B. Arterial thrombosis
C. Thromboangiitis obliterans
D. Thrombophlebitis
Explanations
(u) A. See B for explanation.
(c) B. Arterial thrombosis has occurred and is evidenced by the loss of the popliteal and dorsalis
pedis pulse. This is a surgical emergency. Venous occlusion and thrombophlebitis do not result
in loss of arterial pulse.
(u) C. See B for explanation.
(u) D. See B for explanation.
Ref: (8)

71. Diagnosis/Cardiology
A 48 year-old male with a known history of hypertension is brought to the ED complaining
of headache, general malaise, nausea and vomiting. The patient currently takes nifedipine
(Procardia)90mg XL every day and atenolol (Tenormin) 50 mg every day. Vital signs reveal
temperature 98.6F, pulse 72/minute, respiratory rate 20/minute, and the blood pressure is
168/120 mmHg. BP reading taken every 15 minutes from the time of admission reveal the
systolic to run from 176 to 186 mmHg and the diastolic to run from 135 to 150 mmHg.
Physical examination reveals papilledema bilaterally. There are no renal bruits noted. The
EKG is normal. Based upon this presentation, what is the most likely diagnosis?
Answers
A. Meningitis
B. Secondary hypertension
C. Pseudotumor cerebri
D. Malignant hypertension
Explanations
(u) A. See D for explanation.
(u) B. See D for explanation.

36

(u) C. Pseudotumor cerebri presents with papilledema, but not hypertension and is more common
in young females.
(c) D. Malignant HTN is characterized by diastolic reading greater than 140 mm Hg with evidence
of target organ damage.
Ref: (8)

72. Diagnosis/Cardiology
A 55 year-old male is seen in follow-up for a complaint of chest pain. Patient states that he
has had this chest pain for about one year now. The patient further states that the pain is
retrosternal with radiation to the jaw. "It feels as though a tightness, or heaviness is on
and around my chest". This pain seems to come on with exertion however, over the past
two weeks he has noticed that he has episodes while at rest. If the patient remains nonactive the pain usually resolves in 15-20 minutes. Patient has a 60-pack year smoking
history and drinks a martini daily at lunch. Patient appears overweight on inspection.
Based upon this history what is the most likely diagnosis?
Answers
A. Acute myocardial infarction
B. Prinzmetal variant angina
C. Stable angina
D. Unstable angina
Explanations
(u) A. Pain does not resolve in an acute MI, it gradually gets worse.
(u) B. Pain typically occurs at rest is one of the hallmarks of Prinzmetal variant angina. This
patient has just started to develop pain at rest.
(u) C. Pain in stable angina is relieved with rest and usually resolves within 10 minutes. Stable
angina does not have pain at rest.
(c) D. Pain in unstable angina is precipitated by less effort than before or occurs at rest.
Ref: (27)

73. Diagnosis/Cardiology
A 60 year-old male is brought to the ED complaining of severe onset of chest pain and
intrascapular pain. The patient states that the pain feels as though "something is ripping
and tearing". The patient appears shocky; the skin is cool and clammy. The patient has
an impaired sensorium. Physical examination reveals a loud diastolic murmur and
variation in blood pressure between the right and left arm. Based upon this presentation
what is the most likely diagnosis?
Answers
A. Aortic dissection
B. Acute myocardial infarction
C. Cardiac tamponade
D. Pulmonary embolism
Explanations
(c) A. The scenario presented here is typical of an ascending aortic dissection. In an acute
myocardial infarction the pain builds up gradually. Cardiac tamponade may occur with a
dissection into the pericardial space; syncope is usually seen with this occurrence. Pulmonary
embolism is usually associated with dyspnea along with chest pain.
(u) B. See A for explanation.

37

(u) C. See A for explanation.


(u) D. See A for explanation.
Ref: (27)

74. Diagnosis/Cardiology
A 42 year-old male is brought into the ED with a complaint of chest pain. The pain comes
on suddenly without exertion and lasts anywhere from 10-20 minutes. The patient has
experienced this on three previous occasions. Today the patient complains of lightheadedness with the chest pain lasting longer. Vital signs T-99.3F oral, P-106/minute and
regular, R-22/minute, BP 146/86 mm Hg. EKG reveals sinus rhythm with a rate of 100.
Intervals are PR = 0.06 seconds, QRS = 0.12 seconds. A delta wave is noted in many
leads. Based upon this information what is the most likely diagnosis?
Answers
A. Sinus tachycardia
B. Paroxysmal supraventricular tachycardia
C. Wolff-Parkinson-White syndrome
D. Ventricular tachycardia
Explanations
(u) A. See C for explanation.
(u) B. See C for explanation.
(c) C. Wolff-Parkinson-White syndrome hallmarks on EKG include a shorten PR interval, widened
QRS, and delta waves. Sinus tachycardia has a normal PR interval and no delta waves. PSVT
usually has a retrograde P wave or it may be buried in the QRS complex.
(u) D. Ventricular tachycardia has a widened QRS as it originates in the ventricles.
Ref: (8)

75. Diagnosis/Cardiology
A 63 year-old male is admitted to the hospital with an exacerbation of COPD. The
electrocardiogram shows an irregularly, irregular rhythm at a rate of 120/minute with at
least three varying P wave morphologies. These electrocardiogram findings are most
suggestive of
Answers
A. atrial fibrillation.
B. multifocal atrial tachycardia.
C. atrioventricular junctional rhythm.
D. third degree heart block.
Explanations
(u) A. Atrial fibrillation is an irregularly, irregular rhythm with no definable P waves.
(c) B. Multifocal atrial tachycardia is seen most commonly in patients with COPD.
Electrocardiogram findings include an irregularly, irregular rhythm with a varying PR interval and
various P wave morphologies (Three or more foci).
(u) C. Atrioventricular junctional rhythm is an escape rhythm, because of depressed sinus node
function, with a ventricular rate between 40-60/minute.
(u) D. Third degree heart block presents with a wide QRS at a rate less than 50/minute and
blocked atrial impulses.
Ref: (27)

38

76. Diagnosis/Dermatology
A 56 year-old, right hand dominant, carpenter presents to your clinic complaining of a
prolonged bruise under his left thumbnail. He states that he first noticed it one year ago.
Physical examination reveals a nontender left thumb with a 6 mm macular lesion located
under the distal nail bed. It is mixed dark brown and black in color, with irregular borders.
The most likely diagnosis is
Answers
A. lentigo.
B. trauma.
C. melanoma.
D. nevus.
Explanations
(u) A. Lentigos are typically uniform in color with well-demarcated borders.
(u) B. If the lesion was from trauma, it should have resolved well before one year.
(c) C. Acral lentiginous melanoma may occur on the palm, sole, nail bed, or mucus membrane.
This lesion is suspicious for a melanoma due to its irregular borders, being variegated in color,
and its size. A biopsy is required and will insure the diagnosis.
(u) D. A nevus usually has regular, well-demarcated borders.
Ref: (8)

77. Diagnosis/Dermatology
A mother brings in her 2 year-old child stating that the child has had a 3-day history of a
nonproductive cough, thick copious rhinorrhea, conjunctivitis, and a fever to 103 degrees.
Physical examination reveals a well-hydrated child, with numerous 1-2 mm white papules
on both buccal mucosa, normal heart and breath sounds. This presentation is most
consistent with early
Answers
A. rubeola.
B. rubella.
C. varicella.
D. streptococcal pharyngitis
Explanations
(c) A. Rubeola (measles) is characterized by cough, coryza, and conjunctivitis, along with a fever
as a prodrome. Koplik spots appear prior to the onset of the typical erythematous, maculopapular
rash and are pathognomonic for rubeola.
(u) B. See A for explanation.
(u) C. See A for explanation.
(u) D. See A for explanation.
Ref: (8)

78. Diagnosis/Endocrinology
A 30 year-old female complains of fatigue, weakness, diminished appetite, weight loss,
and syncope. She denies fever, chest or abdominal pain, palpitations, changes in bowel
patterns or sleep patterns. Physical examination reveals a thin female, BP 90/65 mmHg,
and pulse 80 beats per minute. Pulmonary, cardiovascular, abdominal, and neurologic
exam are without abnormalities. Areas of brown and bronze hyperpigmentation are noted

39

on her elbows and the creases of her hands. Which of the following is the most likely
diagnosis?
Answers
A. Addison's disease
B. Cushing's disease
C. Anorexia nervosa
D. Porphyria
Explanations
(c) A. Addisons disease (adrenal insufficiency) would account for all her symptoms, the
hypotension, and the hyperpigmentation of the skin.
(u) B. Cushings disease, the presence of an ACTH-producing adenoma, is characterized by
central obesity, hypertension, moon facies, purple striae, and glucose intolerance.
(u) C. Anorexia nervosa may explain the weakness, weight loss, hypotension, and syncope,
however, a normal pulse rate would be an unexpected finding along with the hyperpigmentation.
(u) D. Porphyria presents acutely with anxiety, depression, disorientation, and insomnia.
Ref: (8)

79. Diagnosis/Endocrinology
A 72 year-old female is being evaluated for recurrent kidney stones. Physical examination
reveals no abnormal findings. Laboratory findings show elevated calcium and decreased
phosphate levels. Which of the following is the most likely diagnosis?
Answers
A. Pheochromocytoma
B. Adrenal insufficiency
C. Hyperparathyroidism
D. Breast cancer
Explanations
(u) A. Pheochromocytoma may lead to hypercalcemia but the patient does not have any signs or
symptoms suggestive of pheochromocytoma, such as hypertension, headache, profuse sweating,
or weight loss.
(u) B. Adrenal insufficiency, Addison's disease, would reveal, in addition to the hypercalcemia,
anorexia, nausea and vomiting, weight loss, and cutaneous hyperpigmentation, none of which are
evident in this patient.
(c) C. The majority of patients with hyperparathyroidism are asymptomatic. Recurrent
nephrolithiasis may be one of the presentations of primary hyperparathyroidism. Measurement of
parathyroid levels would be the initial laboratory test for the evaluation of hypercalcemia.
(a) D. Hypercalcemia may be the earliest manifestation of a malignancy and this must be
investigated. Most often the signs and symptoms of a malignancy will cause the patient to seek
medical care. Malignancy is the second leading cause of hypercalcemia, behind
hyperparathyroidism.
Ref: (8)

80. Diagnosis/Endocrinology
A 38 year-old male presents to your clinic complaining of increasing constant headaches
and progressive loss of peripheral vision. His medical and family history is unremarkable.
Physical examination reveals bitemporal hemianopsia but is otherwise without any
abnormalities. Which of the following is the most likely diagnosis?

40

Answers
A. Aneurysm involving the circle of Willis
B. Migraine headache
C. Multiple sclerosis
D. Pituitary tumor
Explanations
(u) A. An aneurysm involving the circle of Willis would result in CN III palsy. This would be a rare
finding.
(u) B. Although a migraine headache may produce visual field defects, these defects would remit
upon resolution of the migraine. It would also be unusual to have the scotomas occur bilaterally.
(u) C. Optic neuritis associated with multiple sclerosis presents with decreased visual acuity,
dimness, or color desaturation in the central visual field. It would not affect the periphery.
(c) D. A pituitary tumor would account for the headaches and the loss of the peripheral vision in
both visual fields. As the tumor grows, the optic chiasm will be compressed by the tumor.
Ref: (8)

81. Diagnosis/ENT/Ophthalmology
A 23 year-old graduate student presents with sudden onset of severe dizziness, with
nausea and vomiting for the past couple of hours. She denies hearing loss or tinnitus.
She has had a recent cold. Which of the following is the most likely diagnosis?
Answers
A. Mnires disease
B. Vestibular neuronitis
C. Benign positional vertigo
D. Vertebrobasilar insufficiency
Explanations
(u) A. Mnires disease is associated with hearing loss, tinnitus, and vertigo that lasts from
seconds to hours.
(c) B. Vestibular neuronitis or labyrinthitis presents with vertigo, nausea, and vomiting, but not
hearing loss or tinnitus. It is related to viral URIs, and develops over several hours, with
symptoms worse in the first day, with gradual recovery over several days.
(u) C. Benign positional vertigo occurs with changes in position, especially rapid movements of
the head. Nausea may occur, but vomiting is not significant.
(u) D. Vertebrobasilar insufficiency is usually accompanied by brain stem findings, such as
diplopia, dysarthria, or dysphagia, and is not common in this age group.
Ref: (12)

82. Diagnosis/ENT/Ophthalmology
A 4 year-old boy presents with purulent, foul-smelling nasal discharge for three days. He
has not had any other symptoms of respiratory illness, cough, wheeze, or fever. His
activity level and appetite has been normal. On exam, he is afebrile. TMs have normal
light reflex, canals are clear. Left nare is clear; there is considerable amount of purulent
exudate from the right nare, and a bright reflection of light is noticed. Oropharynx is
without inflammation or exudate. Neck is supple, without lymphadenopathy. Lungs are
clear, with equal breath sounds and no wheezing. Heart has regular rhythm without
murmurs. Which of the following is the most likely diagnosis?
Answers
A. Viral URI

41

B. Acute sinusitis
C. Allergic rhinitis
D. Nasal foreign body
Explanations
(u) A. Viral URI does not present with foul-smelling nasal discharge.
(u) B. Acute sinusitis may present with purulent nasal discharge, but the observation of a bright
light reflection suggests a foreign body.
(u) C. Allergic rhinitis is seasonal, associated with sneezing and other allergy-related symptoms.
(c) D. Nasal foreign body is suggested by unilateral nasal obstruction or discharge .
Ref: (6)

83. Diagnosis/ENT/Ophthalmology
A 59 year-old male complains of flashing lights behind my eye followed by sudden loss
of vision, stating that it was like a curtain across my eye. He denies trauma. He takes
Glucophage for his diabetes mellitus and atenolol for his hypertension. He has no other
complaints. On funduscopic exam, the retina appears to be out of focus. Which of the
following is the most likely diagnosis?
Answers
A. Central retinal vein occlusion
B. Retinal artery occlusion
C. Retinal detachment
D. Hyphema
Explanations
(u) A. Central retinal vein occlusion causes painless, variable loss of vision. Exam shows retinal
hemorrhages in all quadrants and edema of the optic disk.
(u) B. Retinal artery occlusion presents with sudden, painless loss of vision. Exam shows pale
retina with normal macula, seen as a cherry-red spot.
(c) C. Patients with retinal detachment frequently complain of flashes of light or floaters that occur
during traction on the retina as it detaches. This is followed by loss of vision. In small
detachments, the retina may appear out of focus, but with larger detachments, a retinal fold may
be identified.
(u) D. Hyphema is usually associated with trauma, and is a collection of blood in the anterior
chamber.
Ref: (30)

84. Diagnosis/ENT/Ophthalmology
A 64 year-old woman complains of headache and left eye pain for about a day. She says it
started yesterday as a dull ache and now is throbbing. She also complains of nausea and
vomiting, which she attributes to the popcorn she ate at the movie theater yesterday
afternoon. On exam, the left pupil is mid-dilated and nonreactive. The cornea is hazy. A
ciliary flush is noted. Which of the following is the most likely diagnosis?
Answers
A. Migraine headache
B. Temporal arteritis
C. Acute glaucoma
D. Retinal artery occlusion
Explanations

42

(u) A. Migraine headache does not present with eye findings.


(u) B. Temporal arteritis presents with headache and systemic symptoms of fever, myalgias,
anorexia, and tenderness over the temporal artery.
(c) C. Acute glaucoma often presents with abdominal complaints that may delay diagnosis.
Findings of ciliary flush, mid-dilated and nonreactive pupil, and hazy cornea in a patient with
severe eye pain are consistent with acute angle closure glaucoma.
(u) D. Retinal artery occlusion presents with sudden, painless, severe loss of vision. There are
no systemic symptoms.
Ref: (28)

85. Diagnosis/Gastrointestinal/Nutritional
A 76 year-old female presents to the ED with the worst abdominal pain in her life. The pain
began following a large meal and is located periumbilically. Although she is writhing in
pain, she does not have an exacerbation of the pain on palpation of the abdomen. She has
a history of coronary artery disease, asthma, and atrial fibrillation. Which of the following
is the most likely diagnosis?
Answers
A. Toxic megacolon
B. Mesenteric thrombosis
C. Fulminant hepatitis
D. Acute diverticulitis with perforation
Explanations
(u) A. Toxic megacolon is a complication seen with ulcerative colitis or electrolyte abnormalities in
which the bowel loses its tone.
(c) B. This patient is at risk for mesenteric ischemia due to advanced age, atherosclerosis and
atrial fibrillation. This is the classic presentation for this condition with pain out of proportion to
physical examination findings.
(u) C. Fulminant hepatitis is most likely to cause malaise, loss of taste, lethargy, and right upper
quadrant pain.
(u) D. Acute diverticulitis with perforation will cause left lower quadrant abdominal pain and
severe pain on palpation due to the peritonitis that occurs from the perforation of bowel contents.
Ref: (8)

86. Diagnosis/Gastrointestinal/Nutritional
A 25 year-old Physician Assistant student is preparing for the national board certification
examination. The student stays up all night and is so busy studying that he forgets to eat
or drink. When he arrives at the test site, he is jaundiced but denies abdominal pain or
tenderness. The jaundice disappears with rest and eating. Laboratory testing reveals an
elevation in the indirect bilirubin in a fasting state but normal test results in a nonfasting
state. What is the most likely diagnosis?
Answers
A. Ehlers-Danlos syndrome
B. Laennecs cirrhosis
C. Chronic hepatitis infection
D. Gilberts disease
Explanations
(u) A. Ehlers-Danlos syndrome is a disease affecting the connective tissues of the body.

43

(u) B. Cirrhosis may involve an increase in various liver function tests but these are not related to
fasting and non-fasting results.
(u) C. Chronic hepatitis may cause jaundice but the results of the liver function tests are not
based upon fasting and non-fasting states.
(c) D. This is an inherited, benign condition resulting in elevations in the indirect bilirubin
concentration. There are no long-term liver abnormalities associated with this condition.
Ref: (8)

87. Diagnosis/Gastrointestinal/Nutritional
A patient is hospitalized with a change in mental status. Examination reveals that he is
unable to maintain dorsiflexion of the wrists after pronating his arms in front of his body.
Which of the following is the most likely diagnosis?
Answers
A. Cocaine overdose
B. Hyperthyroidism
C. Hepatic encephalopathy
D. Parkinsons Disease
Explanations
(u) A. Tremor and agitation are part of acute cocaine intoxication, not asterixis.
(u) B. Hyperthyroidism causes a fine resting tremor, not asterixis.
(c) C. This is the description for asterixis that is seen with hepatic encephalopathy, uremia, and
carbon dioxide narcosis.
(u) D. Parkinsons Disease has resting tremor, rigidity, akinesia, and postural hypotension, not
asterixis.
Ref: (8)

88. Diagnosis/Gastrointestinal/Nutritional
A patient is found to have enlargement of both parotid glands. He is also found to have
failure to thrive with anorexia, weight loss, weakness, and fatigue. On examination vital
signs are BP 135/82 mmHg, pulse 74/minute, and respirations 18/minute. Physical
examination is unremarkable. His hematocrit is 45%, BUN is 15 mg/dl and serum
creatinine is 0.8 mg/dl. Which of the following is the most likely diagnosis?
Answers
A. Hepatorenal syndrome
B. Cirrhosis of the liver
C. Addisons disease
D. Vitamin B12 deficiency
Explanations
(u) A. Patients with hepatorenal syndrome have renal failure that occurs following liver failure.
Although these kidneys would function normally if transplanted into a normal host, they lose their
function as a result of liver impairment.
(c) B. This is the description of a patient who has classic clinical manifestations of cirrhosis of the
liver and liver failure. Jaundice may also be seen if the liver has an inability to metabolize
bilirubin.
(u) C. Addisons disease occurs from adrenal failure to produce glucocorticoids and
mineralocorticoids. The main manifestations are hypotension and skin hyperpigmentation.
(u) D. Vitamin B12 deficiency usually presents with peripheral neuropathy and other neurological
signs, not wasting.

44

Ref: (8)

89. Diagnosis/Hematology
A 60-year-old presents with fatigue and splenomegaly. CBC reveals the following: WBC24,000/microliter, Hgb- 13.5 g/dl, Hct- 40%, MCV- 87 fL, MCHC- 34 g/dl, MCH- 28 pg, and
platelets- 380,000/mL. The differential reveals neutrophils- 11%, lymphocytes- 80%,
monocytes- 8%, and basophils- 1%. What is the most likely diagnosis?
Answers
A. Acute lymphocytic leukemia
B. Acute myelogenous leukemia
C. Chronic lymphocytic leukemia
D. Chronic myelogenous leukemia
Explanations
(u) A. Acute lymphocytic leukemia is more common in children and presents with blasts in the
peripheral blood.
(u) B. Acute myelogenous leukemia presents with pancytopenia and presence of blasts in the
peripheral blood.
(c) C. Chronic lymphocytic leukemia presents with a WBC count greater than 20,000/microliter
and absolute lymphocyte count of greater than 5000/microliter.
(u) D. Chronic myelogenous leukemia presents with elevated WBC count, marked left shift in the
myeloid series of cells, and positive for Philadelphia chromosome.
Ref: (8)

90. Diagnosis/Neurology
A 54 year-old male smoker presents to the clinic complaining of frequent vague
headaches with associated vomiting that awaken him from sleep occasionally and have
been present upon awakening for about two weeks. The headache typically resolves
about an hour into his morning routine. The patient is afebrile. What is the most likely
cause of this patients headaches?
Answers
A. Cluster headaches
B. Depression
C. Glioblastoma
D. Giant cell arteritis
Explanations
(u) A. Cluster headaches can awaken patients, but are not usually "vague".
(u) B. See C for explanation.
(c) C. Morning headaches associated with vomiting are indicative of increased intracranial
pressure and raise concern of a CNS tumor such as a glioblastoma.
(u) D. Giant cell arteritis presents in the older patient with headache in the temporal region and
loss of vision.
Ref: (27)

91. Diagnosis/Neurology
A 28 year-old female presents to the clinic complaining of a prickly sensation that
started bilaterally in her feet two days ago and difficulty walking. She now has the

45

dysesthesia from her mid-thigh down to her toes. On physical examination she has
diminished pain and temperature sensation, absent reflexes, loss of proprioception in her
legs bilaterally, and muscle strength is 1+/5+ in the lower extremities and 5+/5+ in the
upper extremities. What is the most likely diagnosis?
Answers
A. Guillain-Barr syndrome
B. Multiple sclerosis
C. Myasthenia gravis
D. Spinal cord compression
Explanations
(c) A. The pattern of sensory, motor and reflex findings is consistent with the pathophysiology of
peripheral nerve demyelination that occurs in Guillain-Barr syndrome.
(u) B. Multiple sclerosis does not present as a symmetrical ascending paralysis.
(u) C. Patients with myasthenia gravis tend to have intermittent symptoms that affect proximal
and extraocular muscles most notably and it also lacks sensory involvement.
(u) D. Although the exact type of cord transection can alter the pattern of motor and sensory
findings a patient with spinal cord compression who is not in spinal shock would have
hyperreflexia instead of areflexia.
Ref: (27)

92. Diagnosis/Neurology
A 51 year-old female presents to the clinic complaining of intermittent sharp pain that
originates at the corner of her mouth and radiates toward her ipsilateral eye. She notes
"everything makes it worse" including touching the area, talking and eating. What is the
most likely diagnosis?
Answers
A. Bell's palsy
B. Cluster headache
C. Post-herpetic neuralgia
D. Trigeminal neuralgia
Explanations
(u) A. See D for explanation.
(u) B. See D for explanation.
(u) C. See D for explanation.
(c) D. This is the classic presentation for trigeminal neuralgia (tic douloureux).
Ref: (27)

93. Diagnosis/Obstetrics/Gynecology
A 30 year-old presents with persistent vaginal discharge and vulvar pruritus. The
discharge is profuse, frothy, greenish, and foul smelling. pH of the vagina is 6.0. Which of
the following is the most likely diagnosis?
Answers
A. Vulvovaginal candidiasis
B. Bacterial vaginosis
C. Trichomoniasis
D. Atrophic vaginitis

46

Explanations
(u) A. Vulvovaginal candidiasis presents with a thick, curd-like discharge and vulvar pruritus.
(u) B. Bacterial vaginosis presents with malodorous, gray-white discharge. The pH is typically
5.0-5.5.
(c) C. Trichomoniasis presents with vulvar pruritus and a profuse, frothy, greenish, foul-smelling
vaginal discharge with a pH usually exceeding 5.0.
(u) D. Atrophic vaginitis is usually without discharge, but presents with vaginal dryness.
Ref: (9)

94. Diagnosis/Obstetrics/Gynecology
A 30-week pregnant patient presents with sudden onset of profuse, painless vaginal
bleeding. Which of the following is the most likely diagnosis?
Answers
A. Abruptio placentae
B. Uterine rupture
C. Placenta previa
D. Disseminated intravascular coagulation
Explanations
(u) A. Abruptio placentae presents with abdominal pain and vaginal bleeding.
(u) B. Uterine rupture presents with vaginal bleeding or hematuria with suprapubic pain and
tenderness.
(c) C. Placenta previa presents with sudden, painless, profuse bleeding in the third trimester.
(u) D. Disseminated intravascular coagulation presents with systemic signs of bleeding and
thrombosis and typically presents at the time of delivery.
Ref: (9)

95. Diagnosis/Obstetrics/Gynecology
A 25 year-old female presents with constant premenstrual pelvic pain. She also notes
dysmenorrhea and dyspareunia. Which of the following is the most likely diagnosis?
Answers
A. Uterine leiomyoma
B. Endometrial polyps
C. Ovarian cysts
D. Endometriosis
Explanations
(u) A. Leiomyomas of the uterus may present with abnormal uterine bleeding, but typically do not
present with any symptoms.
(u) B. Endometrial polyps present with menorrhagia and intermenstrual and premenstrual
bleeding, pain is not typical.
(u) C. Ovarian cysts present with adnexal mass and acute pain upon rupture.
(c) D. Endometriosis presents with premenstrual pelvic pain, dysmenorrhea, and dyspareunia.
Ref: (9)

96. Diagnosis/Orthopedics/Rheumatology
A 32 year-old male presents with an acute onset of pain and swelling to his left ankle. On
physical exam the ankle is warm, swollen and erythematous. Evaluation of the synovial

47

fluid reveals only leukocytosis with a low glucose. Which of the following is the most
likely diagnosis?
Answers
A. Gout
B. Pseudogout
C. Acute rheumatic fever
D. Septic arthritis
Explanations
(u) A. Gout and pseudogout are excluded by the failure to find crystals on synovial fluid analysis.
(u) B. See A for explanation.
(u) C. Acute rheumatic fever commonly involves multiple joints.
(c) D. Leukocytosis and a low synovial glucose are indicative of septic arthritis.
Ref: (27)

97. Diagnosis/Orthopedics/Rheumatology
A 32-year-old male presents with migratory arthralgias and profound malaise and fatigue.
He states that one week ago he returned from a hunting trip in Pennsylvania. He is also
complaining of a lesion on his left thigh that he noticed about 3 days ago. Physical exam
reveals a large annular lesion with a bright red outer border and partial central clearing.
Which of the following is the most likely diagnosis?
Answers
A. Rheumatoid arthritis
B. Kawasaki disease
C. Lyme disease
D. Nongonococcal arthritis
Explanations
(u) A. Rheumatoid arthritis is a symmetrical arthritis that commonly affects the proximal
interphalangeal and metacarpophalangeal joints. It is not associated with the rash of erythema
migrans.
(u) B. Kawasaki disease is an acute febrile, multisystem disease of children. It is characterized
by unresponsiveness to antibiotics, nonsuppurative cervical adenitis, and changes in the skin and
mucous membranes such as edema, erythema of the lips and palms, and desquamation of the
skin of the fingertips.
(c) C. After an incubation period of 3 to 32 days, erythema migrans develops at the site of the tick
bite. Within days or weeks after the onset of erythema migrans the patients develop a severe
headache, mild stiffness of the neck, migratory musculoskeletal pain, arthralgias and profound
malaise and fatigue.
(u) D. Nongonococcal arthritis occurs in patients with an underlying predisposition such as
rheumatoid arthritis. The common presentation is involvement of a single joint.
Ref: (8)

98. Diagnosis/Orthopedics/Rheumatology
A 22 year-old male presents with pain along the medial tibia. The pain initially began
towards the end of soccer practice but now it is present earlier on. Physical exam reveals
pain to palpation over the posterior tibialis muscle body. What is the most likely
diagnosis?
Answers

48

A. Shin splint
B. Stress fracture
C. Osgood-Schlatter disease
D. Patellofemoral pain syndrome
Explanations
(c) A. Shin splints cause pain over the posterior tibialis muscle body as opposed to discrete pain
over the tibia with a stress fracture.
(u) B. See A for explanation.
(u) C. Osgood-Schlatter disease is an injury occurring at the insertion of the patellar tendon on
the tibial tuberosity in a younger age group.
(u) D. Patellofemoral pain syndrome is the most common cause of chronic anterior knee pain,
more commonly seen in females.
Ref: (6)

99. Diagnosis/Orthopedics/Rheumatology
A patient who demonstrates pain on the radial aspect of the wrist with abrupt ulnar
movements while the thumb is flexed into the closed palm most likely has
Answers
A. carpal tunnel syndrome.
B. radial tunnel syndrome.
C. tenosynovitis.
D. gamekeeper's thumb.
Explanations
(u) A. Carpal tunnel is diagnosed by a positive Tinel's or Phalen's maneuver.
(u) B. Radial syndrome is demonstrated by simultaneously extending the patient's wrists and
fingers while the examiner passively flexes the patient's long finger, which causes pain.
(c) C. Tenosynovitis is diagnosed using Finkelstein maneuver. The patient's thumb is placed in
the palm of the hand and the wrist is abruptly deviated to the ulnar aspect of the wrist, causing
pain on the radial aspect.
(u) D. Gamekeeper's thumb is the most common injury to the metacarpophalangeal collateral
ligaments, causing a sprain.
Ref: (25)

100. Diagnosis/Psychiatry/Behavioral Medicine


A 20 year-old male presents to the ED with complaints of palpitations and agitation, which
developed suddenly while attending a party. On examination, the patient is moderately
agitated and tremulous. Vital signs include a pulse of 110/minute and regular; respiratory
rate 22/minute and blood pressure 160/92 mmHg. Skin is diaphoretic and pupils are
dilated. Which of the following is the most likely diagnosis?
Answers
A. Heroin use
B. Cocaine use
C. Scombroid ingestion
D. Alcohol intoxication
Explanations
(u) A. Opioids, such as heroin produce euphoria, drowsiness and constricted pupils. Severe
intoxication causes bradycardia, respiratory arrest, and hypotension.

49

(c) B. Cocaine, as well as amphetamines, leads to a clinical picture of increased sympathetic


stimulation and dilated pupils.
(u) C. Clinical findings of scombroid include flushing, reversal of hot and cold sensations and
hives.
(u) D. Alcohol, as well as barbiturates and sedative-hypnotic agents, cause central nervous
system depression, leading to bradycardia and hypotension.
Ref: (27)

101. Diagnosis/Psychiatry/Behavioral Medicine


A 53 year-old man with a history of hypertension is being treated with atenolol (Tenormin).
He currently presents complaining of chronic fatigue, insomnia, decreased appetite, and
difficulty concentrating for the past 3 weeks. His wife also notes that he no longer goes
bowling with his friends and has lost interest in any sexual intimacy. Physical
examination is unremarkable. Which of the following is the most likely diagnosis?
Answers
A. major depression
B. dysthymic disorder
C. atypical depression
D. drug-induced depression
Explanations
(c) A. Diagnostic criteria for a major depressive disorder include a loss of pleasure in usual
activities, vegetative or physical changes (poor appetite, loss of energy), and cognitive changes
such as difficulty in concentrating.
(u) B. A dysthymic disorder is a chronic depressive disorder whose symptoms are milder, but
longer lasting (> 2 years) than those in a major depressive episode.
(u) C. Atypical depression is characterized by hypersomnia, overeating, lethargy, and rejection
sensitivity, which are not present in this case.
(u) D. While beta-blockers may cause fatigue and sleep disturbances, they do not cause a
depressive disorder.
Ref: (27)

102. Diagnosis/Psychiatry/Behavioral Medicine


A 56 year-old man is admitted to a hospital unit for evaluation of rectal bleeding and
weight loss. He has a strong family history of cancer. Soon after admission, a barium
enema is scheduled. The patient refuses the "prep" because he fears x-ray radiation. He
states he has had previous x-rays, but becomes frightened at the thought of an x-ray and
"can't face it." The most likely diagnosis is
Answers
A. hysterical personality.
B. dissociative state.
C. conversion reaction.
D. phobic neurosis.
Explanations
(u) A. Hysterical personality consists of multiple physical complaints referable to several other
organ systems.
(u) B. Dissociative state is precipitated by an emotional event that produces fugue, amnesia,
somnambulism, multiple personality, and depersonalization.

50

(u) C. Conversion reaction is characterized by physical symptoms in parts of the body related to
psychic conflict.
(c) D. Phobic neurosis is a phobic ideation of displacement where the patient transfers feelings of
anxiety from the object to one that can be avoided.
Ref: (27)

103. Diagnosis/Pulmonology
You are called to the nursery to see a male infant, born by uncomplicated vaginal delivery.
He weighs 2,600 grams and has one deep crease on the anterior third of each foot.
Respirations are 88 breaths/minute with expiratory grunting and intercostals retractions.
He is cyanotic on room air and becomes pink when placed in 60% oxygen. Chest x-ray
shows atelectasis with air bronchograms. Which of the following is the most likely
diagnosis?
Answers
A. neonatal pneumonia
B. congenital heart disease
C. hyaline membrane disease
D. chronic lung disease of prematurity
Explanations
(u) A. While tachypnea, grunting, retractions and cyanosis may be signs of neonatal pneumonia,
they are primarily late findings of progressive respiratory distress and would not be seen
immediately at the time of delivery. A chest x-ray in pneumonia would also most commonly
reveal an infiltrate or effusion.
(u) B. While congenital heart disease may present with cyanosis, the chest x-ray will reveal a
cardiac abnormality, such as cardiomegaly.
(c) C. Hyaline membrane disease is the most common cause of respiratory distress in the
premature infant. The infant typically presents with tachypnea, cyanosis and expiratory grunting.
A chest x-ray reveals hypoexpansion and air bronchograms.
(u) D. Chronic lung disease of prematurity is a complication in about 20% of infants with hyaline
membrane disease. It is defined as respiratory symptoms, oxygen requirement and chest x-ray
abnormalities at 1 month of age so it cannot be diagnosed at this time in this newborn.
Ref: (14)

104. Diagnosis/Pulmonology
A 15 year-old male presents with a 1 week history of hacking non-productive cough, low
grade fever, malaise and myalgias. Examination is unremarkable except for a few
scattered rhonchi and rales upon auscultation of the chest. The chest x-ray reveals
interstitial infiltrates and a cold agglutinin titer was negative. Which of the following is the
most likely diagnosis?
Answers
A. acute bronchitis
B. viral pneumonia
C. mycoplasma pneumonia
D. pneumococcal pneumonia
Explanations
(u) A. While the patients clinical symptoms of dry cough and rhonchi support this diagnosis, the
chest x-ray would be normal or only show a mild increase in bronchovascular markings, not
infiltrates.

51

(c) B. The patients clinical symptoms as well as chest x-ray findings and negative cold agglutinin
titer are most consistent with viral pneumonia.
(a) C. While the gradual onset of symptoms suggest mycoplasma, the negative cold agglutinin
titer makes this less likely.
(u) D. In older children the signs and symptoms of pneumococcal pneumonia are similar to an
adult and consist of an abrupt onset of cough, fever and chills. The chest x-ray would reveal a
lobar consolidation, not interstitial, picture.
Ref: (14)

105. Diagnosis/Pulmonology
A 32 year-old African American female presents with complaints of a gradual worsening of
exertional dyspnea associated with a mild dry cough. She has tried various cough
preparations on her own without any significant relief. Her examination is essentially
unremarkable. A chest x-ray reveals the presence of bilateral hilar adenopathy. Which of
the following is the most likely diagnosis?
Answers
A. silicosis
B. sarcoidosis
C. tuberculosis
D. mycoplasma pneumonia
Explanations
(u) A. Most patients with silicosis are asymptomatic, but in late stages it may present with
dyspnea. A chest x-ray finding highly suggestive of silicosis is the calcification of the periphery
of the hilar lymph nodes (eggshell calcification).
(c) B. Patients with sarcoidosis present with an insidious onset of dyspnea that may be
associated with malaise and fever. Incidence is the highest in the African American population
and females are affected more frequently than males. Typical chest x-ray findings include
bilateral hilar and right paratracheal lymphadenopathy.
(u) C. The most common pulmonary complaint of tuberculosis is chronic cough associated with
fatigue, weight loss, fever and night sweats. While dyspnea may be present it is a sign of
extensive disease. In addition to hilar lymphadenopathy on chest x-ray, primary tuberculosis
would also reveal small homogeneous infiltrates and segmental atelectasis.
(u) D. While development of mycoplasma pneumonia is gradual, symptoms commonly include not
only cough and dyspnea, but also fever, headache and sore throat. On exam most patients will
also have rales and wheezes. A chest x-ray reveals diffuse interstitial infiltrates.
Ref: (27)

106. Diagnosis/Pulmonology
A 53 year-old female status post abdominal hysterectomy 3 days ago suddenly develops
pleuritic chest pain and dyspnea. On exam she is tachycardic and tachypneic with rales in
the left lower lobe. A chest x-ray is unremarkable and an EKG reveals tachycardia. Which
of the following is the most likely diagnosis?
Answers
A. atelectasis
B. pneumothorax
C. pulmonary embolism
D. myocardial infarction
Explanations

52

(u) A. Small atelectasis is commonly asymptomatic, while large atelectasis may produce signs of
dyspnea and cough. Exam reveals absence of breath sounds in the area involved and dullness
to percussion. A chest x-ray would reveal various findings dependent on the location of the
atelectasis, but would not be normal.
(u) B. While a pneumothorax commonly presents with pleuritic chest pain and dyspnea, exam
would reveal the presence of diminished breath sounds and hyperresonance on the involved
side. A chest x-ray would reveal presence of a pleural line on the expiratory chest x-ray.
(c) C. Risk factors for pulmonary embolism include advanced age, surgery and prolonged
bedrest. While the diagnosis of pulmonary embolism is difficult due to nonspecific clinical
findings, the most common symptoms include pleuritic chest pain and dyspnea associated with
tachypnea. Chest x-ray and EKG are usually normal.
(u) D. While a myocardial infarction usually presents with dyspnea, the chest pain is not usually
pleuritic in nature. An EKG would commonly reveal ST segment changes, which would be
consistent with ischemia or infarct.
Ref: (27)

107. Diagnosis/Urology/Renal
A 65 year-old male with a 60 pack-year smoking history presents with painless hematuria
for two days. He also complains of frequency and dysuria. He denies a history of recent
upper respiratory tract infection. Which of the following is the most likely diagnosis?
Answers
A. Bladder cancer
B. Wegener's granulomatosis
C. IgA nephropathy
D. Benign prostatic hypertrophy
Explanations
(c) A. Bladder cancer is associated with smoking and presents with painless hematuria.
(u) B. Wegener's granulomatosis disease involves the kidneys and the lungs. Renal signs
include hematuria, red blood cells casts, and proteinuria.
(u) C. IgA nephropathy typically presents after an upper respiratory tract infection and presents
with hematuria and proteinuria.
(u) D. BPH presents with voiding symptoms such as hesitancy, straining, weak stream and
postvoid dribbling.
Ref: (8)

108. Diagnosis/Urology/Renal
A patient on acetazolamide for glaucoma complains of increasing shortness of breath.
The patient denies cough, chest pain, or fever. Physical examination is unremarkable
except for an increased respiratory rate of 30/minute. Labs include Na 132 mEq/L, K 5.6
mEq/L, Cl 120 mEq/L, and CO2 10 mEq/L. Arterial blood gas results are pH 7.18, pO2 98
mmHg, pCO2 22 mmHg, and HCO3 8 mEq/L. Based upon these results, what is the
diagnosis?
Answers
A. Metabolic alkalosis
B. Metabolic acidosis
C. Respiratory alkalosis
D. Respiratory acidosis
Explanations

53

(u) A. See B for explantation.


(c) B. Acetazolamide inhibits carbonic anhydrase, which inhibits bicarbonate ion regeneration,
with resulting normal anion-gap, hyperchloremic acidosis. Respiratory compensatory
mechanisms lead to hyperventilation.
(u) C. See B for explantation.
(u) D. See B for explantation.
Ref: (28)

109. Diagnosis/Infectious Diseases


A patient presents with a rash six days after returning from a camping and hiking trip in
the woods. The rash is macular and first appeared on the ankles and then the rest of the
lower extremities. On exam the physician assistant notes lesions on the soles of the feet
and the trunk. Which of the following is the most likely diagnosis?
Answers
A. Rocky mountain spotted fever
B. Typhoid fever
C. Lyme disease
D. Q fever
Explanations
(c) A. Rocky mountain spotted fever presents with a macular rash on the wrists, ankles,
extremities, and trunk. After 5 days the rash appears on the palms and soles.
(u) B. The rash of typhoid fever is a faint, salmon-colored, maculopapular rash. The rash is noted
primarily on the trunk and chest. The palms and soles are spared.
(u) C. Lyme disease presents with a red macule or papule rash, which expands slowly with
central clearing at the site of the tick bite. Palms and soles are spared.
(u) D. Cattle, sheep, and goats transmit Q fever. The rash is non-specific and not seen on the
palms or soles.
Ref: (8)

110. Health Maintenance/Cardiology


A 72 year-old female is being discharged from the hospital following an acute anterolateral
wall myocardial infarction. While in the hospital the patient has not had any dysrhythmias
or hemodynamic compromise. Which of the following medications should be a part of her
discharge medications?
Answers
A. Warfarin (Coumadin)
B. Captopril (Capoten)
C. Digoxin (Lanoxin)
D. Furosemide (Lasix)
Explanations
(u) A. Warfarin is not indicated since there is no role for anticoagulation in this patient.
(c) B. ACE inhibitors have been shown to decrease left ventricular hypertrophy and remodeling to
allow for a greater ejection fraction.
(u) C. The patient does not have any dysrhythmias so Lanoxin is not indicated.
(u) D. The patient does not have any hemodynamic compromise or indicators of CHF.
Ref: (27)

54

111. Health Maintenance/Cardiology


A 44 year-old male with a known history of rheumatic fever at age 7 and heart murmur is
scheduled to undergo a routine dental cleaning. The murmur is identified as an opening
snap murmur. Patient has no known drug allergies. What should this patient receive for
antibiotic prophylaxis prior to the dental cleaning?
Answers
A. This patient does not require antibiotic prophylaxis for a routine dental cleaning.
B. This should receive Pen VK 250 mg p.o. QID for 10 days after the procedure.
C. This patient should receive Amoxicillin 3.0 gms. p.o. 1 hour before the procedure and then 1.5
gm. 6 hours after the procedure.
D. This patient should receive Erythromycin 250 mg QID for 1 day before the procedure and then
10 days after the procedure.
Explanations
(h) A. See C for explanation.
(u) B. See C for explanation.
(c) C. These are the current recommendations from the American Heart Association if the patient
is not allergic to penicillin.
(u) D. See C for explanation.
Ref: (12)

112. Health Maintenance/Cardiology


A 36 year-old female presents for a refill of her oral contraceptives. She admits to smoking
one pack of cigarettes per day. She should be counseled with regard to her risk of
Answers
A. venous thrombosis.
B. varicose veins.
C. atherosclerosis.
D. peripheral edema.
Explanations
(c) A. Women over age 35 who smoke are at increased risk for the development of venous
thrombosis.
(u) B. Varicose veins are the result of pressure overload on incompetent veins and not due to the
use of oral contraceptives.
(a) C. The defined risks of atherosclerosis includes smoking, but does not include the use of oral
contraceptives.
(u) D. There is no relationship between the use of oral contraceptives and the development of
peripheral edema.
Ref: (27)

113. Health Maintenance/Cardiology


A 68 year-old female comes to the office for an annual physical examination. Her past
medical history is significant for a 40-pack year cigarette smoking history. She takes no
medications and has not been hospitalized for any surgery. Family medical history
reveals that her mother is living, age 87, in good health without medical problems. Her
father is deceased at age 45 from a motor vehicle crash. She has two siblings that are
alive and well. From this information, how many identifiable risk factors for
cardiovascular heart disease exist in this patient?

55

Answers
A. 0
B. 1
C. 2
D. 3
Explanations
(u) A. See C for explanation.
(u) B. See C for explanation.
(c) C. This patient has 2 identifiable risk factors based upon the information provided. These
include her age 68 and her history of cigarette smoking.
(u) D. See C for explanation.
Ref: (27)

114. Health Maintenance/Dermatology


A person with atopic dermatitis should be advised to
Answers
A. avoid cutaneous irritants.
B. take hot water baths or showers.
C. use a high potency glucocorticoid on skin after bathing.
D. begin a prophylactic antibiotic.
Explanations
(c) A. Avoidance of cutaneous irritants, such as wool and other rough clothing, is the cornerstone
of therapy for atopic dermatitis.
(u) B. Patients should bathe in warm, tepid water, not hot water.
(u) C. Topical glucocorticoids of low or medium potency in a cream or ointment base should be
used on the skin immediately after bathing.
(u) D. Prophylactic antibiotics are not indicated in the treatment of atopic dermatitis.
Ref: (8)

115. Health Maintenance/Endocrinology


The most effective method to prevent diabetic retinopathy is
Answers
A. routine, nondilated eye exams.
B. laser photocoagulation.
C. intensive glycemic control.
D. use of ACE inhibitors.
Explanations
(u) A. Nondilated eye exams are not adequate to properly detect diabetic retinopathy.
(u) B. Laser photocoagulation is used in preserving vision once diabetic retinopathy has taken
place, but it is not preventative of the disease itself.
(c) C. The most effective method to prevent diabetic retinopathy is through intensive glycemic
control.
(u) D. Although ACE inhibitors help in preventing diabetic nephropathy, they have no effect on
retinopathy.
Ref: (8)

56

116. Health Maintenance/ENT/Ophthalmology


Which of the following is the most common cause of adult blindness in the USA?
Answers
A. Hypertension
B. Diabetes mellitus
C. Macular degeneration
D. Retinal artery occlusion
Explanations
(u) A. See B for explanation.
(c) B. Diabetes is the leading cause of blindness in the US, and is usually due to diabetic
retinopathy. Up to 15% of type 1 diabetics and up to 7% of type 2 diabetics become legally blind.
(u) C. See B for explanation.
(u) D. See B for explanation.
Ref: (8)

117. Health Maintenance/ENT/Ophthalmology


The most common cause of conductive hearing loss is
Answers
A. otosclerosis.
B. cholesteatoma.
C. impacted cerumen.
D. chronic serous otitis media.
Explanations
(u) A. See C for explanation.
(u) B. See C for explanation.
(c) C. The most common cause of conductive hearing loss is impacted cerumen.
(u) D. See C for explanation.
Ref: (12)

118. Health Maintenance/Gastrointestinal/Nutritional


In order to prevent the hepatic complications associated with hemochromatosis, which of
the following is used early in the course of therapy?
Answers
A. Penicillamine
B. Recurrent paracentesis
C. Albumin infusions
D. Recurrent phlebotomy
Explanations
(u) A. Penicillamine is a chelating agent that may be used in the treatment of Wilsons disease in
which there is a deficiency of the copper-binding protein ceruloplasmin resulting in impaired
copper excretion into the bile.
(u) B. Paracentesis may be performed in patients with tense ascites, which can symptomatically
improve the patient but does not alter long-term prognosis.
(u) C. Patients with long-standing liver failure have decreased albumin levels but this is not a
treatment normally performed for hemochromatosis.

57

(c) D. Hemochromatosis is an autosomal recessive inherited disorder that causes cirrhosis,


diabetes, and bronze pigmentation to the skin due to the abnormal accumulation of iron in
tissues. Intensive phlebotomy is the treatment of choice until the iron overload is corrected.
Ref: (8)

119. Health Maintenance/Gastrointestinal/Nutritional


Which of the following is associated with an increased risk of developing esophageal
cancer?
Answers
A. <i>Helicobacter pylori</i>
B. Familial polyposis
C. Chronic gastric reflux
D. Hepatitis C
Explanations
(u) A. <i>Helicobacter pylori</i> is associated with an increased risk of gastric cancer.
(u) B. Familial polyposis is associated with an increased risk of colon cancer.
(c) C. Chronic gastric reflux is associated with an increased risk of esophageal cancer.
(u) D. Hepatitis C is associated with an increased risk of hepatocellular carcinoma.
Ref: (8)

120. Health Maintenance/Hematology


A patient with multiple myeloma should be immunized against which of the following
organisms?
Answers
A. Streptococcus pneumoniae
B. Hepatitis C virus
C. Listeria monocytogenes
D. Epstein Barr virus
Explanations
(c) A. Patients with multiple myeloma are prone to infections with encapsulated organisms such
as Streptococcus pneumoniae and Haemophilus influenzae.
(u) B. See A for explanation.
(u) C. See A for explanation.
(u) D. See A for explanation.
Ref: (8)

121. Health Maintenance/Neurology


A 45 year-old man presents for a routine appointment. He tells you his mother and father
have both had ischemic strokes in their 70s. He does not smoke. His blood pressure is
128/80 mmHg, pulse 78/minutes and regular, respiratory rate of 12/minute. What diagnostic
studies would you order to further evaluate this patients risk of stroke?
Answers
A. Electrocardiogram
B. Fasting lipid profile
C. Carotid Doppler ultrasound

58

D. MRI with gadolinium


Explanations
(u) A. The main risk factor assessed by ECG is atrial fibrillation and this patients regular pulse
confirms he is currently not in atrial fibrillation
(c) B. Hyperlipidemia is a known risk factors for stroke that can be modified with treatment.
(u) C. The patient does not have signs or symptoms of carotid stenosis at this point.
(u) D. An MRI with gadolinium would be useful in evaluating for the presence of Berry aneurysms,
but the history of ischemic strokes does not raise the concern of an aneurysm.
Ref: (27)

122. Health Maintenance/Obstetrics/Gynecology


Which of the following prenatal vitamins has been shown to decrease the risk of neural
tube defects?
Answers
A. Riboflavin
B. Niacin
C. Thiamine
D. Folic acid
Explanations
(u) A. See D for explanation.
(u) B. See D for explanation.
(u) C. See D for explanation.
(c) D. Folic acid given daily has been shown to effectively reduce the risk of neural tube defects.
It should be started 1-3 months prior to pregnancy.
Ref: (9)

123. Health Maintenance/Obstetrics/Gynecology


According to the United States Preventive Services Task Force, screening mammography
for females without risk factors should be started at what age?
Answers
A. 35
B. 40
C. 45
D. 50
Explanations
(u) A. See B for explanation.
(c) B. The United States Preventive Services Task Force recommends that women should
consider screening mammography at age 40 in order to decrease morbidity and mortality from
the disease.
(u) C. See B for explanation.
(u) D. See B for explanation.
Ref: (12)

59

124. Health Maintenance/Orthopedics/Rheumatology


The most important preventive medicine recommendation for patients with osteoarthritis
is which of the following?
Answers
A. Start an exercise program
B. Brace the affected joint
C. Rest the joint
D. Inject steroids monthly
Explanations
(c) A. Patients with osteoarthritis who exercise are able to maintain range of motion, strengthen
periarticular muscles, and improve physical fitness.
(u) B. Bracing or resting of the affected joint in a patient with osteoarthritis may limit joint motion
and restrict mobility, which may negatively impact the disease.
(u) C. See B for explanation.
(u) D. Steroid injection into the affected joint may decrease pain but injections are limited to not
more than three per year.
Ref: (8)

125. Health Maintenance/Orthopedics/Rheumatology


Bone mass measurement should be considered in all women by what age?
Answers
A. 30 35
B. 40 45
C. 50 55
D. 60 65
Explanations
(u) A. See D for explanation.
(u) B. See D for explanation.
(u) C. See D for explanation.
(c) D. According to the National Osteoporosis Foundation, all women should have a bone mass
measurement by age 60 65.
Ref: (8)

126. Health Maintenance/Psychiatry/Behavioral Medicine


Rates of alcohol use in the adolescent are reportedly higher in:
Answers
A. families with rigorous parental monitoring
B. Japanese families
C. adoptive children whose biological parent is an alcoholic
D. mood disorders in a grandparent
Explanations
(u) A. Families with the lowest measures of parental supervision, children initiated alcohol,
tobacco, and drug use earlier than children from families with more supervision.
(u) B. Alcoholism has been reported to be lower among Japanese families.
(c) C. There is a three-to fourfold increase in risk for alcohol dependence in adopted children
whose biological parents are alcohol dependents.

60

(u) D. There is no correlation suggesting that mood disorders in a grandparent increases the risk
of alcohol dependency in the adolescent.
Ref: (8)

127. Health Maintenance/Pulmonology


A 17 year-old male who is trying out for the track team notes excessive coughing with
chest tightness when running. Which of the following is the most appropriate preventive
agent for this patient?
Answers
A. Albuterol inhaler (Proventil)
B. Inhaled corticosteroids
C. Aminophylline (Theo-Dur)
D. Ipratropium (Atrovent)
Explanations
(c) A. Albuterol is a beta-2 agonist that results in bronchodilation that makes this a useful agent in
a patient with exercise-induced asthma when used just prior to exercise.
(u) B. Inhaled corticosteroids are effective in exercise-induced asthma but are not acute acting
and due to the side effect profile, they are not first line agents.
(u) C. Aminophylline is not used as a first-line agent as a bronchodilator for patients with
exercise-induced asthma.
(u) D. Ipratropium main use is with suppression of mucous secretions and this is not a component
of the exercise-induced asthma patient.
Ref: (8)

128. Health Maintenance/Pulmonology


A 62 year-old female is admitted to a nursing home during an outbreak of influenza. In
review of her records, you note that she did not receive the flu vaccine this year. Which of
the following is the most appropriate drug of choice for influenza prophylaxis in this
patient?
Answers
A. Ciprofloxin (Cipro)
B. Zanamivir (Relenza)
C. Clarithromycin (Biaxin)
D. Alpha-2b interferon (Avonex)
Explanations
(u) A. Ciprofloxin is indicated for postexposure prophylaxis of anthrax.
(c) B. Either zanamivir or oseltamivir are indicated for prophylactic use against influenza A or B.
(u) C. Clarithromycin is indicated for prophylaxis against disseminated Mycobacterium avium
complex.
(u) D. Alpha-2b interferon is indicated for treatment of several disorders, such as chronic hepatitis
B & C, but has no role in prophylactic treatment of any condition.
Ref: (27)

129. Health Maintenance/Pulmonology


Which of the following is an independent risk factor for development of a mesothelioma?

61

Answers
A. Cigarette smoking
B. Asbestos exposure
C. Radon gas exposure
D. Chronic obstructive lung disease
Explanations
(u) A. There has not been any evidence of association between cigarette smoking and the
development of mesothelioma.
(c) B. Studies confirm the association of asbestos exposure to the development of mesothelioma.
(u) C. After cigarette smoking, radon gas is the second most common risk factor for development
of bronchogenic lung cancer, not mesothelioma.
(u) D. Chronic obstructive lung disease is associated with an increased risk of bronchogenic lung
cancer, not mesothelioma.
Ref: (27)

130. Health Maintenance/Urology/Renal


Patients with recurrent urinary stone disease should be educated to maintain a diet
restricted in
Answers
A. sodium and protein.
B. carbohydrates and fat.
C. bran.
D. fluids.
Explanations
(c) A. Increased sodium intake will increase sodium and calcium excretion and increase
monosodium urate saturation. Protein also increases calcium, oxalate and uric acid excretion.
All these factors can lead to stone formation.
(u) B. Carbohydrates and fat do not have any impact on urinary stone disease.
(u) C. Bran significantly lowers urinary calcium, reducing risk for stone recurrence.
(u) D. Increased fluids is important in reducing stone recurrence.
Ref: (27)

131. Health Maintenance/Infectious Diseases


A recent Asian immigrant presents to the clinic for an employment physical examination
before starting work at a local hospital. The patient has a history of receiving a bacille
Calmette-Guerin (BCG) vaccination. Screening for tuberculosis should include which of
the following tests?
Answers
A. Bronchoalveolar lavage
B. Tine test
C. Chest x-ray
D. No screening is needed
Explanations
(h) A. Bronchoalveolar lavage should not be used as a screening test for tuberculosis. This is an
invasive test that carries high risk without additional benefit.
(u) B. Tine test is not indicated for use in screening of TB in any circumstance.

62

(c) C. Chest x-ray is the test of choice in patients where the PPD test is not indicated or in highrisk individuals.
(u) D. This patient has recently emigrated from a possible endemic region and should be
screened for tuberculosis.
Ref: (8)

132. Clinical Intervention/Cardiology


Following an acute anterolateral myocardial wall infarction two days ago, a patient
suddenly develops hemodynamic deterioration without EKG changes occurring. What
complication can explain this scenario?
Answers
A. Free wall rupture
B. CVA
C. Atrial fibrillation
D. Sick sinus syndrome
Explanations
(c) A. Free wall rupture is a complication that occurs within 72 hours of infarction. It is seen
mainly in Q wave transmural and lateral wall infarctions.
(u) B. See A for explanation.
(u) C. See A for explanation. Atrial fibrillation would have EKG evidence of irregularly, irregular
rate and rhythm.
(u) D. Sick sinus syndrome would have EKG evidence of decreased rate and loss of P waves.
Ref: (8)

133. Clinical Intervention/Cardiology


A 48 year-old male with a history of coronary artery disease and two myocardial
infarctions complains of shortness of breath at rest and 2-pillow orthopnea. His oxygen
saturation is 85% on room air. The patient denies any prior history of symptoms. The
patient denies smoking. Results of a beta-natriuretic peptide (BNP) are elevated. What
should be your next course of action for this patient?
Answers
A. Send him home on 20 mg furosemide (Lasix) p.o. every day and recheck in one week
B. Send him home on clarithromycin (Biaxin) 500 mg p.o. BID and recheck in 1 week
C. Admit to the hospital for work up of left ventricular dysfunction
D. Admit to the hospital for work up of pneumonia
Explanations
(h) A. See C for explanation.
(h) B. See C for explanation.
(c) C. An elevated BNP is seen in a situation where there is increased pressure in the ventricle
during diastole. This is representative of the left ventricle being stretched excessively when a
patient has CHF. Sending a patient home would be inappropriate in this case.
(u) D. See C for explanation.
Ref: (27)

63

134. Clinical Intervention/Cardiology


A 48 year-old male presents to the ED with complaints of chest pressure, dyspnea on
exertion, and diaphoresis that has been present for the last one hour. Electrocardiogram
reveals normal sinus rhythm at 92/minute along with ST segment elevation in leads V3-V5.
Initial cardiac enzymes are normal. What is the next most appropriate step in the
management of this patient?
Answers
A. Coronary artery revascularization
B. Admission for medical management
C. Administer lidocaine
D. Administer nitrates
Explanations
(c) A. The standard of care for the management of acute ST-segment elevation MI is coronary
artery revascularization. This patient is diagnosed with an ST-segment elevation MI based upon
his history and EKG findings. Cardiac enzymes are normal because of the early presentation of
this patient.
(u) B. Although this patient will be admitted to the hospital, this patient needs to have acute
management of the myocardial infarction without delay.
(h) C. Prophylactic lidocaine has been shown to increase morbidity and mortality from acute MI
when used in this setting.
(u) D. Although pain control is a goal for patients with acute MI, it is not the essential medication
that will impact this patient's care to the greatest degree.
Ref: (27)

135. Clinical Intervention/Cardiology


An unresponsive patient is brought to the ED by ambulance. He is in ventricular
tachycardia with a heart rate of 210 beats/min and a blood pressure of 70/40 mmHg. The
first step in treatment is to
Answers
A. administer IV adenosine.
B. DC cardiovert.
C. administer IV lidocaine.
D. apply overdrive pacer.
Explanations
(u) A. Adenosine is used to treat PSVT.
(c) B. The first step in treatment of unstable ventricular tachycardia with a pulse is to cardiovert
using a 100 J countershock.
(u) C. See B for explanation.
(u) D. Overdrive pacing is indicated in Torsades de Pointes.
Ref: (28)

136. Clinical Intervention/Dermatology


An elderly woman presents to your clinic complaining of unilateral facial pain and painful
lesions. She also complains of blurred vision in the ipsilateral eye. On examination she
has several vesicles on an erythematous base, some of the lesions with crusts. They are
distributed in a dermatomal pattern and involve the skin overlying the maxillary region and
the tip of her nose. Which of the following is the next most appropriate intervention in the
care of this patient?

64

Answers
A. KOH prep
B. Culture for bacteria
C. Referral to an ophthalmologist
D. Application of corticosteroids
Explanations
(u) A. A KOH prep examining for fungal elements is not indicated in this case.
(u) B. While bacterial infection may occur along with the viral infection, a bacterial culture is not
usually employed.
(c) C. Immediate referral to an ophthalmologist is needed when herpes keratitis is suspected, as
in this case. A fluorescein stain of the eye might reveal the typical dendritic corneal lesion.
(h) D. Application of corticosteroids may cause proliferation of the virus and should only used by
an ophthalmologist
Ref: (28)

137. Clinical Intervention/Dermatology


A 28 year-old female with diabetes mellitus type 2 sustains a partial thickness burn to her
left upper arm and her chest when hot grease spilled on her at home. The burn to her arm
is circumferential and the estimated total body surface burned is 18%. She has no
allergies. The most appropriate treatment of this patient would include
Answers
A. outpatient application of silver sulfadiazine.
B. debridement of all intact blisters.
C. IV cefazolin (Ancef, Kefzol).
D. transfer to a burn center.
Explanations
(u) A. Although treatment may include silver sulfadiazine dressings, this patient should not be
treated as an outpatient. Also see D for explanation.
(u) B. Debridement of intact blisters remains controversial, however many authorities recommend
leaving intact blisters intact and only debride ruptured blisters.
(u) C. If used in the care of a burn patient, the antibiotic selected should have activity against
Pseudomonas and <i>S. aureus</i>. Cefazolin does not have any antipseudomonal activity.
(c) D. Reasons for transfer to a burn center include a partial thickness burn covering greater than
10% of total body surface area. In addition, burns in patients with pre-existing medical conditions,
such as diabetes, that could complicate their management, prolong recovery, or affect their
outcome, is also a reason for transfer to a burn center.
Ref: (29)

138. Clinical Intervention/Endocrinology


A diabetic patient returns for follow-up of non-fasting blood work done at a local health
fair. The total cholesterol is 230 mg/dl. Which of the following is the appropriate next
step?
Answers
A. Reassurance
B. Give advice on diet and exercise
C. Obtain a fasting HDL and LDL lipid measurement.
D. Start an HMG Co-A reductase inhibitor (statin)

65

Explanations
(u) A. See C for explanation.
(u) B. See C for explanation.
(c) C. A patient with an elevated total cholesterol needs further evaluation through a fasting total
lipid profile including LDL and HDL.
(u) D. See C for explanation.
Ref: (27)

139. Clinical Intervention/Endocrinology


Radioactive iodine is most successful in treating hyperthyroidism that results from
Answers
A. Grave's disease.
B. subacute thyroiditis.
C. Hashimoto's thyroiditis.
D. papillary thyroid carcinoma.
Explanations
(c) A. Radioactive iodine is an excellent method to destroy overactive thyroid tissue of Grave's
disease.
(u) B. Radioactive iodine is ineffective in subacute thyroiditis due to the thyroid's low uptake of
iodine.
(u) C. Radioiodine uptake is low in Hashimoto's thyroiditis and is often transient.
(u) D. Papillary thyroid carcinoma is a common thyroid malignancy and must be treated by a
thyroidectomy.
Ref: (8)

140. Clinical Intervention/ENT/Ophthalmology


An 18 year-old college student took her goggles off in the chemistry lab while she was
washing her glassware from an experiment she had just completed. She thinks the beaker
had contained sodium hydroxide, and it splashed into her right eye. She rinsed her eye
out for about five minutes in the lab. Which of the following is the most appropriate first
step?
Answers
A. Check visual acuity
B. Flush the eye with two liters saline
C. Check for corneal damage with fluorescein
D. Instill a topical anesthetic to facilitate examination
Explanations
(u) A. Delaying irrigation of the eye to assess vision or corneal damage is inappropriate as
damage to the eye may occur during the delay.
(c) B. Any chemical injury to the eye may result in severe injury and loss of sight, and is a true
emergency. The eye should be irrigated copiously with at least two liters of saline, and pH
checked to determine when the chemical has been cleared.
(u) C. See A for explanation.
(u) D. Topical anesthetic may be used to facilitate irrigation with a Morgan lens, but see A for
explanation.
Ref: (28)

66

141. Clinical Intervention/ENT/Ophthalmology


There is considerable debate about the use of tympanostomy tubes in the management of
recurrent otitis media in children. Tympanostomy tube placement has been proven to
Answers
A. improve hearing.
B. prevent mastoiditis.
C. prevent recurrence of effusion.
D. prevent delayed language development.
Explanations
(c) A. Hearing is improved with tympanostomy tubes by eliminating middle ear effusion when the
tubes are functioning properly.
(u) B. Mastoiditis is prevented by early treatment of otitis media with antibiotics.
(u) C. Effusion can occur even with tympanostomy tubes in place.
(u) D. Tympanostomy tubes have not been proven to prevent delayed language development.
Ref: (6)

142. Clinical Intervention/ENT/Ophthalmology


A 66 year-old male presents with epistaxis of two hours duration. He has been unable to
stop the bleeding at home. He has a history of hypertension and cardiovascular disease.
On exam the vital signs are T-38C, P-74/minute, RR-20/minute, and BP-180/110 mmHg.
Bleeding is noted from the right nare; the left is essentially clear with normal nasal
mucosa. After applying pressure for 15 minutes, a bilateral anterior pack is placed. You
note persistent bleeding, with blood in the posterior oropharynx. Of the following, the
most appropriate treatment is to remove the packing and
Answers
A. replace with Gelfoam packing.
B. cauterize Kiesselbachs plexus.
C. insert a posterior balloon pack.
D. apply topical solution of lidocaine and epinephrine
Explanations
(u) A. Gelfoam packing, cauterization of Kiesselbachs plexus, and topical lidocaine and
epinephrine are helpful in treating an anterior bleed, but not in a posterior bleed.
(u) B. See A for explanation.
(c) C. A posterior balloon packing is used to treat posterior epistaxis. These patients must be
admitted to the hospital and prompt consultation with an otolaryngologist is indicated.
(u) D. See A for explanation.
Ref: (28)

143. Clinical Intervention/Gastrointestinal/Nutritional


The treatment of choice for patients with celiac disease is which of the following?
Answers
A. Gluten-free diet
B. Small bowel resection
C. Clindamycin
D. Whipple procedure

67

Explanations
(c) A. Celiac disease is a diffuse disease of the small bowel that is caused by immunologic
malfunction. This disease is active only in the presence of gluten, a constituent of wheat.
Avoidance of gluten-containing foods is the treatment of choice.
(u) B. Although celiac disease affects the small bowel (particularly the jejunum), resection is too
aggressive when dietary manipulation is very effective in the management of this disorder.
(u) C. Celiac disease is immune-mediated and is not due to infection so antibiotics are not
needed.
(h) D. Whipple procedure is done for tumors of the head of the pancreas and has no role in the
management of celiac disease.
Ref: (8)

144. Clinical Intervention/Gastrointestinal/Nutritional


A 77 year-old male presents with a two-week history of cough. The patient also complains
of a bad taste in his mouth. He complains of mild midsternal chest pressure but denies
any dyspnea. He denies any dysphagia or history of recent URI. Evaluation of the cardiac
and pulmonary systems, including pulmonary function tests and exercise stress testing, is
normal. Upper endoscopy is normal. Which of the following is the next step in the
evaluation of this patient?
Answers
A. Upper GI study with barium
B. Esophageal pH monitoring
C. CT of the thorax
D. D-xylose test of the stool
Explanations
(u) A. An upper GI study will not add any diagnostic value to this evaluation following the
performance of endoscopy.
(c) B. Esophageal pH monitoring records the pH of the lower esophagus on a 24-hour continuous
basis. This may be the only abnormality that occurs in a patient with GERD who presents with
coughing as the chief complaint.
(u) C. CT scan of the thorax is most helpful in the evaluation of esophageal cancer and not
GERD.
(u) D. D-xylose testing of the stool is part of the evaluation for possible malabsorption. There is no
consideration of malabsorption for this patient.
Ref: (8)

145. Clinical Intervention/Gastrointestinal/Nutritional


The main complication with the use of transjugular intrahepatic portosystemic shunt
(TIPS) procedure is which of the following?
Answers
A. Esophageal varices
B. Worsening of cirrhosis
C. Budd-Chiari syndrome
D. Increased risk of encephalopathy
Explanations
(u) A. TIPS procedures are performed in order to shunt blood away from the liver parenchyma,
which in essence lowers portal pressures lessening the risk for esophageal varices.

68

(u) B. TIPS procedures, since they cause of bypass of the liver parenchyma, result in a lessening
of the blood flow into the liver, which does not cause cirrhosis to progress.
(u) C. Budd-Chiari syndrome is a thrombosis of the hepatic vein. It is not a common complication
of the TIPS procedure.
(c) D. TIPS procedures involve the placement of a stent in the liver in order to shunt blood away
from the portal vein into the hepatic vein, which bypasses the cirrhotic liver parenchyma. Its main
complication is encephalopathy from the accumulation of toxic substances in the brain since the
liver no longer acts as a filter.
Ref: (8)

146. Clinical Intervention/Hematology


It is essential to advise a patient with infectious mononucleosis to
Answers
A. avoid contact sports.
B. continue to rest as much as possible.
C. gargle with salt water for relief of throat pain.
D. rest voice to prevent hoarseness.
Explanations
(c) A. Patients with acute infectious mononucleosis often have enlargement of the spleen.
Contact sports increase the risk for splenic rupture after contact. The other answers are
suggestions that may improve symptoms but do not truly impact the care of the patient.
(u) B. See A for explanation.
(u) C. See A for explanation.
(u) D. See A for explanation.
Ref: (27)

147. Clinical Intervention/Neurology


Which of the following interventions is most effective during the early stages of
Alzheimer's disease?
Answers
A. Frequent change of caregivers in the home
B. Utilization of memory aids, such as post-it notes
C. Encouragement of independent activities, such as driving
D. Emphasis of new learning activities, such as computer training
Explanations
(u) A. Although caregiver burnout is often encountered, consistency is beneficial to a patient
having difficulty with confusion.
(c) B. Memory aids are extremely helpful in assisting Alzheimer patients during the early stages of
the disease.
(u) C. Like our shuttle driver demonstrated, most patients should consider relinquishing their
license.
(u) D. Pleasant activities should be emphasized, while learning new activities may be a source of
frustration.
Ref: (8)

69

148. Clinical Intervention/Neurology


An 82 year-old male is brought to the ED after being found unresponsive in his apartment.
On physical exam his pupils are 7 mm on the right and 3 mm on the left. What would be
your initial choice in managing this patient?
Answers
A. Order a CBC, electrolytes, and toxicology screen
B. Order a CT of the head
C. Hyperventilate the patient
D. Administer streptokinase
Explanations
(u) A. The patient has PE findings that are asymmetric. This is consistent with an anatomical
abnormality and not a systemic disorder.
(c) B. This presentation is consistent with a structural abnormality that will be detected on an
imaging study.
(h) C. Prophylactic hyperventilation is not recommended because cerebral perfusion pressure
can be decreased and cause further injury to the brain.
(h) D. Streptokinase is not indicated due to possibility of a hemorrhagic event.
Ref: (27)

149. Clinical Intervention/Obstetrics/Gynecology


A 35 year-old female presents with a solitary breast mass. Fine needle aspiration reveals
bloody fluid with no malignant cells. What is the next best step in the care of this patient?
Answers
A. Monthly follow-up
B. Excisional biopsy
C. Hormone therapy
D. Repeat fine needle aspiration
Explanations
(h) A. Monthly follow-up is indicated in cases where the mass disappears with aspiration or the
fluid is clear.
(c) B. Excisional biopsy is the next step in cases of bloody fluid, residual mass or thickening.
(u) C. Hormone therapy is indicated in cases of breast cancer that express hormone receptors.
(u) D. Repeat fine needle aspiration is indicated in solid masses which are benign or in which
cytologic findings are inconclusive.
Ref: (9)

150. Clinical Intervention/Obstetrics/Gynecology


What is the primary treatment for an infected Bartholin's duct cyst?
Answers
A. Acyclovir
B. Azithromycin
C. Warm compresses
D. Incision and drainage
Explanations
(u) A. Herpes infections are not a common cause of Bartholin's duct cysts.
(u) B. Antibiotics are reserved for cases having surrounding inflammation.

70

(u) C. Warm compresses are not indicated in the treatment of infected Bartholin's duct cysts.
(c) D. Incision and drainage is the treatment of choice for a Bartholin duct cyst.
Ref: (9)

151. Clinical Intervention/Orthopedics/Rheumatology


A 32 year-old medical transcriptionist presents with burning and tingling in her right wrist
and hand for the past month. On physical exam, Phalens test is positive; however, there
is no atrophy of the thenar eminence. Which of the following is the initial step in
management of this patient?
Answers
A. Wrist splint for 2-6 weeks
B. Corticosteroid injection
C. Surgical referral
D. Darvocet
Explanations
(c) A. The treatment of carpal tunnel syndrome is aimed at relieving the pressure on the median
nerve. This is best accomplished by having the patient wear a wrist splint during the activities
that increase the pressure on the median nerve.
(u) B. Corticosteroid injections and surgery are indicated only after a trial of the wrist splint
provides no relief.
(u) C. See B for explanation.
(u) D. Darvocet has no role in the treatment of carpal tunnel syndrome.
Ref: (27)

152. Clinical Intervention/Orthopedics/Rheumatology


Which of the following is the correct treatment for a Grade II ankle sprain resulting from an
inversion injury?
Answers
A. Corticosteroid injection
B. Rest, ice, compression, elevation
C. Moist heat and a walking cast
D. Surgical intervention
Explanations
(u) A. See B for explanation.
(c) B. The majority of ankle sprains are treated with RICE (rest, ice, compression and elevation).
Corticosteroid injections, moist heat, and a walking cast provide no benefit. Surgical intervention
for repair of a ruptured ligament is only necessary in chronically unstable joints.
(u) C. See B for explanation.
(u) D. See B for explanation.
Ref: (27)

153. Clinical Intervention/Orthopedics/Rheumatology


A 4 year-old boy presents to the ED after sustaining a crush injury to his distal third
phalanx. Physical exam reveals an associated nail bed injury. Which of the following is
the appropriate management?

71

Answers
A. Rest, ice, elevation
B. Immobilize, antibiotics, orthopedics referral
C. Splint for 48 hours, aspirin, ice
D. Surgical referral for amputation of digit
Explanations
(u) A. Rest, ice, and elevation are only palliative measures and not appropriate management
(c) B. Distal phalanx fracture should be immobilized and if there is an associated nailbed injury
the fracture is considered open and the patient should be given antibiotics and follow-up with
ortho in one week.
(u) C. The digit must be immobilized until seen by ortho. Aspirin is not an appropriate analgesic
for children.
(u) D. See B for explanation.
Ref: (28)

154. Clinical Intervention/Psychiatry/Behavioral Medicine


A patient presents within one hour of ingesting 30 tablets of diazepam (Valium). Which of
the following is the most appropriate intervention?
Answers
A. IV naloxone
B. alkalinization of the urine
C. cardiac pacing
D. gastric lavage
Explanations
(u) A. Naloxone (Narcan) is an opioid agonist and would have no effect in a benzodiazepine
overdose.
(u) B. Alkalinization of the urine may be useful in the management of a barbiturate overdose, but
not a benzodiazepine overdose.
(u) C. Cardiac pacing would be useful if there were asystole from the overdose.
(c) D. Gastric lavage, along with the administration of activated charcoal and monitoring of vital
signs and CNS status is the mainstay of therapy in the person who has overdosed on
benzodiazepines. Vomiting should be induced in the person who is not comatose. Flumazenil
(Romazicon), a specific benzodiazepine antagonist, might be used with caution in certain
patients.
Ref: (28)

155. Clinical Intervention/Psychiatry/Behavioral Medicine


The most important initial component of evaluating a patient with depressive illness is
Answers
A. assessment of suicidal risk.
B. assessment of memory function.
C. determination of the presence of hallucinations.
D. determination of the degree of intellectual impairment.
Explanations
(c) A. Two-thirds of all depressed patients contemplate suicide, and 10 to 15% commit suicide.

72

(u) B. Assessment of memory function and degree of intellectual impairment is important during
the evaluation of the depressed patient, as both may be impaired in over 95% of patients. These,
however, are not the most important.
(u) C. Hallucinations are not a symptom of depression.
(u) D. See B for explanation.
Ref: (15)

156. Clinical Intervention/Pulmonology


A 3 month-old male presents with a hoarse cough and thick purulent rhinorrhea for the
past 2 days. The mother noted that yesterday he appeared to get worse and seemed to
have increasing problems breathing and trouble feeding. Examination reveals a
temperature of 100.2 degrees F and respiratory rate of 80/minute with nasal flaring and
retractions. Lung examination reveals a prolonged expiratory phase with inspiratory rales.
He is tachycardic. Pulse oximetry reveals oxygen saturation of 89%. Chest x-ray reveals
hyperinflation with diffuse interstitial infiltrates. Which of the following is the most
appropriate intervention?
Answers
A. antibiotics
B. hospitalization
C. inhaled corticosteroids
D. Racemic epinephrine
Explanations
(u) A. Antibiotics are utilized to treat bacterial, not viral, illnesses.
(c) B. This infant most likely has bronchiolitis. While most cases are mild and can be treated at
home, hospitalization is recommended for infants with hypoxia on room air, moderate tachypnea
with feeding difficulties and marked respiratory distress with retractions. Additionally
hospitalization is recommended for infants less than 2-3 months of age, a history of apnea or an
underlying chronic cardiopulmonary disease.
(u) C. The use of corticosteroids in children with bronchiolitis has not been studied and does not
appear to be helpful.
(u) D. Racemic epinephrine is not indicated in the treatment of bronchiolitis.
Ref: (14)

157. Clinical Intervention/Pulmonology


A 62 year-old male presents with a history of dyspnea on exertion and chronic cough
worse with arising in the mornings. He has a 40-year-pack history of cigarette use. On
examination there is increased AP diameter and decreased breath sounds with a
prolonged expiratory phase. Pulse oximetry reveals an oxygen saturation of 93% on room
air. In addition to smoking cessation, which of the following is an appropriate intervention
at this time?
Answers
A. Home oxygen therapy
B. Maintenance oral steroids
C. Prophylactic antibiotic therapy
D. Recommend influenza and pneumococcal vaccines
Explanations
(u) A. Home oxygen therapy is indicated in COPD patients with an oxygen saturation < or equal
to 88% or a pO2 < or equal to 55 mm Hg taken at rest breathing room air.

73

(u) B. While oral steroids may be utilized in treatment of COPD, they are usually reserved for end
stage disease due to the multiple systemic side effects of prolonged use. In addition only about
10% of patients show any increase in FEV1 and there use should be reserved for patients who
show a 20% or greater improvement in FEV1.
(u) C. Use of antibiotics should be reserved for treatment of acute exacerbations of COPD, acute
bronchitis or documented bacterial infections, not prophylaxis.
(c) D. In addition to smoking cessation, patients may benefit from vaccination against both
influenza and pneumococcal disease.
Ref: (27)

158. Clinical Intervention/Pulmonology


A 21 year-old male presents to the ED with increasing dyspnea and pleuritic chest pain of
sudden onset after getting hit in the left side of the chest during a bar fight. Examination
reveals moderate respiratory distress with absence of breath sounds and hyperresonance
to percussion on the left, with tracheal deviation to the right. Which of the following is the
most appropriate next step?
Answers
A. order a V/Q scan
B. order a chest x-ray
C. administer a sclerosing agent
D. insert large bore needle into left 2nd ICS stat
Explanations
(h) A. A V/Q scan is indicated in suspected cases of pulmonary embolism.
(u) B. Patients in respiratory distress and evidence of a tension pneumothorax, such as tracheal
deviation, should have treatment initiated without waiting on a chest x-ray to be taken.
(h) C. Pleurodesis by administration of a sclerosing agent is indicated in treatment of recurrent,
not traumatic, pneumothorax.
(c) D. Simple aspiration by insertion of a needle into the involved side will decompress the tension
pneumothorax until a chest tube can be inserted.
Ref: (28)

159. Clinical Intervention/Pulmonology


Which of the following is a major contraindication to curative surgical resection of a lung
tumor?
Answers
A. Liver metastases
B. Vagus nerve involvement
C. Non-malignant pleural effusion
D. Chest wall invasion of the tumor
Explanations
(c) A. Distant metastases, except for solitary brain and adrenal metastases are an absolute
contraindication for pulmonary resection. Other absolute contraindications include MI within past
3 months, SVC syndrome due to metastatic tumor, bilateral endobronchial tumor, contralateral
lymph node metastases and malignant pleural effusion.
(u) B. See A for explanation.
(h) C. See A for explanation.
(u) D. See A for explanation.

74

Ref: (31)
160. Clinical Intervention/Urology/Renal
A 32 year-old female patient presents with renal colic and hematuria. The patient has a
long-standing history of unresponsive treatment for urinary tract infections with
documented Proteus species. Urinalysis reveals crystals resembling coffin lids. KUB
reveals a staghorn calculus in the right kidney. Which of the following is the best clinical
intervention?
Answers
A. High fluid intake with a low salt diet
B. A low purine diet
C. Laser lithotripsy
D. Percutaneous nephrolithotomy
Explanations
(u) A. High fluid intake (>3L/day) and a low salt diet is helpful in patients with cystine stones.
(u) B. Low purine diets are instituted in patients who form uric acid stones with hyperuricosuria.
(u) C. Laser lithotripsy is used for removal of ureteral stones via a urethroscope.
(c) D. Percutaneous nephrolithotomy is currently the primary surgical intervention of choice for
struvite stones.
Ref: (8)

161. Clinical Intervention/Urology/Renal


Which of the following is the most appropriate intervention for a stage I testicular
seminoma?
Answers
A. Watchful waiting
B. Chemotherapy initially
C. Orchiectomy and radiation
D. Orchiectomy and chemotherapy
Explanations
(u) A. Surveillance is an option in stage I disease of a nonseminoma testicular tumor.
(u) B. Patients with stage IIC and stage III are treated with chemotherapy.
(c) C. Inguinal orchiectomy followed by retroperitoneal radiation therapy cures about 98% of
patients with stage I seminoma.
(u) D. Chemotherapy is used for later stage tumors (II/III) and followed by surgery in stage III
tumors.
Ref: (8)

162. Clinical Intervention/Infectious Diseases


A 23 year-old presents with a plantar wart on the right foot. Which of the following
treatments is most effective with the fewest side effects?
Answers
A. Laser therapy
B. Surgical removal
C. Salicylic acid plaster
D. Podophyllin

75

Explanations
(u) A. Laser therapy and surgical removal can be used for plantar warts, but there are significant
side effects due to the invasive nature of these treatments.
(u) B. See A for explanation.
(c) C. Salicylic acid plaster is safe and effective for the treatment for plantar warts and has
essentially no side effects.
(u) D. Podophyllin is frequently used for treatment of perianal and genital warts, but not used for
plantar warts.
Ref: (27)

163. Clinical Therapeutics/Cardiology


Which of the following antiarrhythmic drugs can be associated with hyper- or
hypothyroidism following long-term use?
Answers
A. Quinidine
B. Amiodarone
C. Digoxin
D. Verapamil
Explanations
(u) A. See B for explanation.
(c) B. Amiodarone is structurally related to thyroxine and contains iodine, which can induce a
hyper- or hypothyroid state.
(u) C. See B for explanation.
(u) D. See B for explanation.
Ref: (10)

164. Clinical Therapeutics/Cardiology


Which of the following hypertensive emergency drugs has the potential for developing
cyanide toxicity?
Answers
A. Sodium nitroprusside (Nipride)
B. Diazoxide (Hyperstat)
C. Labetalol (Normodyne)
D. Alpha-methyldopa (Aldomet)
Explanations
(c) A. Sodium nitroprusside metabolization results in cyanide ion production. It can be treated
with sodium thiosulfite, which combines with the cyanide ion to form thiocyanate, which is
nontoxic.
(u) B. See A for explanation.
(u) C. See A for explanation.
(u) D. See A for explanation.
Ref: (10)

165. Clinical Therapeutics/Cardiology


Contraindications to beta blockade following an acute myocardial infarction include which
of the following?

76

Answers
A. Third degree A-V block
B. Sinus tachycardia
C. Hypertension
D. Rapid ventricular response to Atrial fibrillation/flutter
Explanations
(c) A. Beta blockade is contraindicated in second and third heart block.
(u) B. Beta blockade has been proven to be beneficial in sinus tachycardia, hypertension and in
atrial fib/flutter with a rapid ventricular response.
(u) C. See B for explanation.
(u) D. See B for explanation.
Ref: (27)

166. Clinical Therapeutics/Cardiology


A 74 year-old male is diagnosed with pneumonia. The physician assistant should ensure
the patient is not on which of the following before starting therapy with clarithromycin
(Biaxin)?
Answers
A. Lisinopril (Zestril)
B. Furosemide (Lasix)
C. Simvastatin (Zocor)
D. Dipyridamole (Persantine)
Explanations
(u) A. See C for explanation.
(u) B. See C for explanation.
(c) C. Statins are known to interact with the macrolides as they may cause prolonged QT interval,
myopathy and rhabdomyolysis.
(u) D. See C for explanation.
Ref: (10)

167. Clinical Therapeutics/Cardiology


According to the recent JNC VII guidelines, a 34 year-old male who has type 1 diabetes
mellitus and hypertension should be started on which type of antihypertensive agent?
Answers
A. Beta-blocker
B. Loop diuretic
C. ACE inhibitor
D. Thiazide diuretic
Explanations
(u) A. Beta blockers could potentially be harmful in a patient with diabetes mellitus. Use a
cardioselective beta-blocker to reduce the incidence of hypoglycemia.
(u) B. See C for explanation.
(c) C. ACE inhibitors are effective in young patients. They are capable of providing protection to
the kidney especially in diabetes mellitus.
(u) D. See C for explanation.

77

Ref: (27)

168. Clinical Therapeutics/Cardiology


Which of the following beta-adrenergic blocking agents has cardioselectivity for primarily
blocking beta-1 receptors?
Answers
A. Propranolol (Inderal)
B. Timolol (Blocadren)
C. Metoprolol (Lopressor)
D. Pindolol (Visken)
Explanations
(u) A. Propranolol and timolol are nonselective beta-adrenergic antagonists.
(u) B. See A for explanation.
(c) C. Metoprolol is selective for beta-1 antagonists
(u) D. Pindolol is an antagonist with partial agonist activity.
Ref: (10)

169. Clinical Therapeutics/Dermatology


Pharmacologic treatment of a cat bite in the person with no allergies consists of which of
the following?
Answers
A. Trimethoprim-sulfamethoxazole (Bactrim)
B. Cephalexin (Keflex)
C. Ceftriaxone (Rocephin)
D. Amoxicillin-clavulanate (Augmentin)
Explanations
(u) A. See D for explanation.
(u) B. See D for explanation.
(u) C. See D for explanation.
(c) D. Amoxicillin-clavulanate (Augmentin) has activity against <i>Pasteurella multocida</i> which
is the causative agent in the majority of cat bite infections.
Ref: (8)

170. Clinical Therapeutics/Endocrinology


A newborn infant exhibits prolonged jaundice, feeding problems, hypotonia, and an
enlarged tongue. Proper treatment in this infant would consist of which of the following?
Answers
A. IV antibiotics
B. Thyroid hormone replacement
C. Hepatitis B immunoglobulin
D. Vitamin B6 supplement
Explanations
(u) A. See Answer C for explanation.

78

(c) B. This scenario is consistent with congenital hypothyroidism. Measurement of TSH or T4


would confirm this and T4 should be given.
(u) C. Hepatitis and sepsis may account for the presence of jaundice, feeding problems, and
hypotonia, but would not result in an enlarged tongue.
(u) D. A deficiency in vitamin B6 may lead to glossitis but would not account for or any of the
other signs.
Ref: (8)

171. Clinical Therapeutics/Endocrinology


Which of the following glucose-lowering agents act by delaying glucose absorption?
Answers
A. Metformin (Glucophage)
B. Acarbose (Precose)
C. Glipizide (Glucotrol)
D. Pioglitazone (Actos)
Explanations
(u) A. Metformin, a biguanide, lowers glucose by decreasing hepatic glucose production and
increased glucose utilization.
(c) B. Alpha-glucosidase inhibitors, such as acarbose, reduce glucose by delaying glucose
absorption.
(u) C. Glipizide and other sulfonylureas work by increasing insulin secretion.
(u) D. Pioglitazone is a thiazolidinedione and decreases insulin resistance and increases glucose
utilization.
Ref: (8)

172. Clinical Therapeutics/ENT/Ophthalmology


A 20 year-old woman presents with a 3-day history of sneezing, watery nasal discharge,
and a nonproductive cough. Her throat was sore for the first 2 days, and she now
complains of fatigue and difficulty breathing because of her "stuffy nose." Which of the
following is most likely to improve this patient's status?
Answers
A. Ascorbic acid
B. Amoxicillin
C. Pseudoephedrine
D. Chlorpheniramine
Explanations
(u) A. Ascorbic acid is of no proven benefit in the treatment of viral URI.
(u) B. The use of antibiotics is inappropriate for viral infections.
(c) C. A decongestant will relieve the nasal congestion and stuffy nose symptoms.
(a) D. An antihistamine may relieve the sneezing, but it may thicken secretions, making them
difficult to clear.
Ref: (8)

173. Clinical Therapeutics/ENT/Ophthalmology


A 12 year-old presents with complaint of both eyes watering. He also complains of
sinus congestion and sneezing for two weeks. On exam vital signs are T-38C, P-

79

80/minute, and RR-20/minute. The eyes reveal mild conjunctival injection bilaterally, clear
watery discharge, and no matting. Pupils are equal, round, and reactive to light and
accommodation. The extraocular movements are intact. The funduscopic exam shows
normal disc and vessels. The TMs are normal and the canals are clear. The nasal mucosa
is boggy, with clear rhinorrhea. Which of the following is the most helpful pharmacologic
agent?
Answers
A. Artificial tears
B. Tobramycin drops
C. Erythromycin ointment
D. Naphazoline (Naphcon-A) drops
Explanations
(u) A. Artificial tears are used for dry eyes.
(u) B. Tobramycin drops and erythromycin ointment are used to treat bacterial infections.
(u) C. See B for explanation.
(c) D. Naphazoline is a topical antihistamine that relieves symptoms of allergic conjunctivitis.
Ref: (12)

174. Clinical Therapeutics/ENT/Ophthalmology


A 13 year-old presents with pain in his right ear and loss of hearing since yesterday. He
has never had an episode like this before. On exam vital signs are T 38C P 70/minute
R 18/minute BP 90/60 mmHg. Neck is supple, without lymphadenopathy. Right tympanic
membrane is not visible; the canal is swollen, with small amount of exudate noted. There
is tenderness of the external ear, especially with gentle traction of the tragus. Left
tympanic membrane is normal, and the canal is clear. Oropharynx is normal. Which of the
following is the most appropriate topical treatment for this patient?
Answers
A. Acetic acid solution
B. Erythromycin solution
C. Cortisporin otic suspension
D. Gentamycin drops
Explanations
(u) A. Acetic acid solution may be used, but the burning sensation associated with it reduces
patient compliance. It is usually used in combination with a topical antimicrobial.
(u) B. Erythromycin solution is not used to treat otitis externa.
(c) C. Cortisporin otic suspension is a combination antimicrobial (Neomycin and Polymyxin B) and
steroid (hydrocortisone) that is effective to use in the treatment of otitis externa. In addition, the
suspension is safe to use in suspected cases of tympanic perforation.
(u) D. Gentamycin drops are not used in the treatment of otitis externa.
Ref: (6)

175. Clinical Therapeutics/ENT/Ophthalmology


A 19 year-old college student complains of a sore throat for over a week, with fever and
general malaise. On exam T-38C P-70/minute R-20/minute BP-110/76 mmHg. The patient is
alert and oriented x 3. The skin is warm, dry, and without rash. The TMs have a normal
light reflex and the canals are clear. The oropharynx is inflamed, with bilaterally enlarged
tonsils, and a small amount of exudate. The neck is supple, with anterior cervical
adenopathy. The lungs are clear. The heart has a regular rhythm without murmurs. The

80

abdomen is soft, nontender and a spleen tip is palpable. The labs reveal a negative rapid
strep screen and positive Monospot. The WBC count is 9,000/microliter with a differential
of 40% atypical lymphocytes, 35% lymphocytes, 5% monocytes, 10% eosinophils, and 10%
neutrophils. Which of the following is the most appropriate treatment?
Answers
A. Penicillin
B. Erythromycin
C. Acetaminophen
D. Acyclovir
Explanations
(u) A. Antibiotics are not indicated in the treatment infectious mononucleosis, or Ebstein-Barr
virus infections.
(u) B. See A for explanation.
(c) C. Aspirin or acetaminophen may be used to treat fever and pain associated with infectious
mononucleosis, or EBV infection.
(u) D. Acyclovir is not approved for use in treatment of EBV, although it is active against the EBV
in vitro and in vivo. It may be used in certain patients with AIDS, but has not been shown to affect
the outcome of EBV in these patients.
Ref: (12)

176. Clinical Therapeutics/Gastrointestinal/Nutritional


Which of the following is considered to be the treatment of choice for the pruritus that
occurs with primary biliary cirrhosis?
Answers
A. Colchicine
B. Atorvastatin (Lipitor)
C. Cholestyramine (Questran)
D. Enalapril (Vasotec)
Explanations
(u) A. Colchicine is used to improve the biochemical abnormalities, which may slow the
progression of the disease.
(u) B. Statins have a role in the management of hyperlipidemia but are not effective in this
disease.
(c) C. Cholestyramine, a bile salt sequestrant, is able to decrease the pruritus that occurs from
the bile stasis and granulomas.
(u) D. ACE inhibitors are effective as antihypertensives and in preserving renal function in those
with proteinuria, but they have no role in the management of primary biliary cirrhosis.
Ref: (8)

177. Clinical Therapeutics/Gastrointestinal/Nutritional


Which of the following medications is most useful in maintaining remission in a patient
with ulcerative colitis?
Answers
A. Oral corticosteroids
B. Corticosteroid enemas
C. Sulfasalazine
D. Macrodantin

81

Explanations
(u) A. Corticosteroids, given orally or via enema preparations, are used for acute flare-up of
ulcerative colitis but are not effective when given as prophylactic agents.
(u) B. See A for explanation.
(c) C. Sulfasalazine, olsalazine, and mesalamine are effective in maintaining remission in patients
with ulcerative colitis.
(u) D. Long-term antibiotic therapy with Macrodantin is used as a prophylactic agent to prevent
urinary tract infections and has no role in the treatment or prevention of ulcerative colitis.
Ref: (8)

178. Clinical Therapeutics/Gastrointestinal/Nutritional


Which of the following is the treatment of choice for patients with Hepatitis C?
Answers
A. Prednisolone
B. Acyclovir and azathioprine
C. Cyclosporine
D. Pegylated interferon and ribavirin
Explanations
(u) A. See D for explanation.
(u) B. See D for explanation.
(u) C. See D for explanation.
(c) D. Pegylated interferon and ribavirin is the only approved treatment for Hepatitis C infection.
Ref: (8)

179. Clinical Therapeutics/Gastrointestinal/Nutritional


A 35 year-old male completed a 2-week course of ciprofloxacin and now complains of
abdominal cramping and diarrhea. Stool is positive for C. difficile toxins. Which of the
following is the drug of choice for treatment?
Answers
A. Amoxicillin
B. Metronidazole
C. Tetracycline
D. Trimethoprim-sulfamethoxazole
Explanations
(u) A. Amoxicillin, tetracycline, and trimethoprim-sulfamethoxazole are not indicated for the
treatment of pseudomembranous colitis.
(c) B. Metronidazole is the drug of choice for the treatment of pseudomembranous colitis.
(u) C. See A for explanation.
(u) D. See A for explanation.
Ref: (8)

180. Clinical Therapeutics/Hematology


A 6 year-old male presents with hemarthrosis of the left knee. Coagulation studies reveal
the following results: PT- 12.5 seconds with an INR: 1.0, aPTT:58 seconds, platelet count:

82

430,000/microliter, and bleeding time:4 minutes. Which of the following is the best
treatment option for this patient?
Answers
A. Desmopressin acetate
B. Corticosteroids
C. Vitamin K
D. Cryoprecipitate
Explanations
(u) A. Desmopressin acetate is indicated in von Willebrand's disease, which presents with a
prolonged bleeding time.
(u) B. Corticosteroids are indicated in immune-mediated thrombocytopenia.
(u) C. Vitamin K deficiency will prolong the PT greater than the aPTT. Vitamin K supplement is
not indicated in this patient.
(c) D. Hemophilia A presents with a prolonged aPTT and normal platelet count and function.
Hemophilia A is treated with factor VIII concentrate or cryoprecipitate.
Ref: (8)

181. Clinical Therapeutics/Neurology


A 72 year-old man with long-standing diabetes mellitus, renal insufficiency and
hypertension presents to the clinic complaining of burning and tingling pain in his feet.
What agent would you prescribe to help control his pain?
Answers
A. Phenobarbital
B. Amitriptyline (Elavil)
C. Celecoxib (Celebrex)
D. Codeine
Explanations
(u) A. Phenobarbital is used in the treatment of seizure disorder.
(c) B. Amitriptyline is one of several preferred agents for management of peripheral neuropathy.
(u) C. Although Cox-2 inhibitors may provide pain relief they are best avoided for treatment of this
chronic disorder in a patient with renal disease.
(h) D. It is best to avoid narcotics in the management of chronic pain if other efficacious
medicines are available.
Ref: (27)

182. Clinical Therapeutics/Neurology


What is the appropriate initial intravenous drug therapy for a patient in status epilepticus?
Answers
A. Midazolam (Versed)
B. Lorazepam (Ativan)
C. Fosphenytoin (Mesantoin)
D. Phenobarbital
Explanations
(u) A. See B for explanation.
(c) B. A benzodiazepine (lorazepam) is first line in the treatment of status epilepticus, followed by
phenytoin or fosphenytoin.

83

(h) C. See B for explanation.


(h) D. This answer is potentially harmful because the patient is currently seizing and lorazepam
is indicated.
Ref: (8)

183. Clinical Therapeutics/Neurology


A 37 year-old female presents to the clinic for follow-up regarding her recently diagnosed
tonic-clonic epilepsy. She reports no seizures or side effects since starting valproate
(Depakote) at her last visit one month ago. What diagnostic study would you order to
monitor this patient's treatment?
Answers
A. Serum amylase
B. Serum creatinine
C. Liver function tests
D. Electroencephalogram
Explanations
(u) A. See C for explanation.
(u) B. See C for explanation.
(c) C. Valproate may be toxic to the liver as well as cause thrombocytopenia.
(u) D. See C for explanation.
Ref: (27)

184. Clinical Therapeutics/Obstetrics/Gynecology


A female patient presents with a vaginal discharge that has a fishy odor. On wet mount
examination of the discharge a few white blood cells and many stippled epithelial cells are
noted. Which of the following is the treatment of choice for this patient?
Answers
A. Nonoxynol-9
B. Ceftriaxone
C. Metronidazole
D. Clotrimazole
Explanations
(u) A. Nonoxynol-9 is a spermicidal agent and not used to treat bacterial vaginosis.
(u) B. Ceftriaxone is used in the treatment of <i>Neisseria gonorrhoeae</i>.
(c) C. Metronidazole is the treatment of choice for bacterial vaginosis. Bacterial vaginosis
presents with a vaginal discharge with a fishy odor and clue cells on wet mount exam.
(u) D. Clotrimazole is used in the treatment of candidiasis.
Ref: (9)

185. Clinical Therapeutics/Obstetrics/Gynecology


A 36-week pregnant patient presents to the ED with hypertension. Physical examination
reveals 2+/4+ edema in the lower extremities with hyperreflexia and clonus bilaterally. A
urinalysis reveals 3+ protein. Which of the following is the best treatment option for this
patient?
Answers

84

A. Magnesium sulfate
B. Nifedipine (Procardia)
C. Diazepam (Valium)
D. Captopril (Capoten)
Explanations
(c) A. Magnesium sulfate is indicated in this patient with preeclampsia to reduce the risk of
seizures.
(h) B. Nifedipine can lead to severe hypotension and should be avoided in this case.
(h) C. Diazepam may cause respiratory depression, hypotonia, and thermoregulatory problems in
the newborn and should be avoided.
(h) D. Captopril and other ACE inhibitors should be avoided in pregnant patients due to side
effects.
Ref: (9)

186. Clinical Therapeutics/Obstetrics/Gynecology


A patient presents with infertility due to chronic anovulation. Laboratory testing reveals a
normal follicle stimulating hormone, estradiol, and prolactin levels. The patient's
progestin challenge test was positive. Which of the following is the drug of choice for the
treatment of infertility in this patient?
Answers
A. Dehydroepiandrosterone sulfate
B. Clomiphene citrate
C. Bromocriptine
D. Human menopausal gonadotropin
Explanations
(u) A. Dehydroepiandrosterone sulfate is an androgen and has no role in stimulating ovulation.
(c) B. Clomiphene citrate is the first drug of choice in patients with infertility due to anovulation
with normal hormone levels.
(u) C. Bromocriptine is used to induce ovulation in patients with pituitary tumors.
(u) D. Human menopausal gonadotropin is used in patients who do not respond to clomiphene or
have a pituitary insufficiency.
Ref: (9)

187. Clinical Therapeutics/Orthopedics/Rheumatology


Which of the following is the antibiotic of choice when treating a patient with osteomyelitis
whose culture demonstrates penicillin-resistant, methicillin sensitive Staphylococcus
aureus?
Answers
A. Cefuroxime
B. Levofloxacin
C. Nafcillin
D. Tobramycin
Explanations
(u) A. See C for explanation.
(u) B. See C for explanation.

85

(c) C. Treatment for osteomyelitis is as follows: Penicillin G is the drug of choice for the treatment
of infections due to penicillin-sensitive staphylococci and streptococci; nafcillin or oxacillin is
preferred for penicillin-resistant, methicillin-sensitive staphylococci. Regimens for infections due
to susceptible gram-negative rods includes cefuroxime and fluoroquinolones. Tobramycin is
effective against Pseudomonas aeruginosa.
(u) D. See C for explanation.
Ref: (8)

188. Clinical Therapeutics/Orthopedics/Rheumatology


Which of the following is the most common side effect of methotrexate?
Answers
A. Hemolysis
B. Cardiomyopathy
C. Stomatitis
D. Diarrhea
Explanations
(u) A. See C for explanation.
(u) B. See C for explanation.
(c) C. The most common side effects of Methotrexate are stomatitis and gastritis. Other side
effects include hepatotoxicity, cytopenia, and interstitial pneumonitis.
(u) D. See C for explanation.
Ref: (27)

189. Clinical Therapeutics/Orthopedics/Rheumatology


A 45 year-old male presents with an acute onset of asymmetric arthritis of the lower
extremities. Further history reveals an episode of urethritis two weeks ago. Physical
exam reveals a swollen tender right knee and left ankle and vesicles on his palms and
soles. Which of the following is the initial treatment of choice in this patient?
Answers
A. Indomethacin
B. Interferon
C. Sulfasalazine
D. Methotrexate
Explanations
(c) A. This patient has reactive arthritis (formerly known as Reiters syndrome). In recent years,
the term has been used to refer to spondyloarthropathies following and enteric or urogenital
infection. Indomethacin is the treatment of choice. Interferon is not indicated in reactive arthritis.
Sulfasalazine and methotrexate are used in patients with persistent reactive arthritis.
(u) B. See A for explanation.
(u) C. See A for explanation.
(u) D. See A for explanation.
Ref: (8)

190. Clinical Therapeutics/Orthopedics/Rheumatology


A 25 year-old female presents with a constellation of symptoms including fatigue, chronic
headaches, and poor sleep. She also complains of aching pain and stiffness involving her

86

entire body. Minor exertion aggravates the pain and increases her fatigue. Which of the
following classes of medications is the most efficacious treatment for this patient's
condition?
Answers
A. Non-steroidal anti-inflammatories
B. Opioids
C. Corticosteroids
D. Antidepressants
Explanations
(u) A. NSAIDS and steroids are ineffective in treating fibromyalgia.
(h) B. Opioids should not be used due to their addiction potential and poor efficacy.
(u) C. See A for explanation.
(c) D. Antidepressants have shown modest efficacy in treating the symptoms of fibromyalgia.
Ref: (27)

191. Clinical Therapeutics/Psychiatry/Behavioral Medicine


Which of the following antipsychotic medications is associated with significant
agranulocytosis but not with extrapyramidal side effects?
Answers
A. Clozapine (Clozaril)
B. Haloperidol (Haldol)
C. Chlorpromazine (Thorazine)
D. Fluphenazine (Prolixin)
Explanations
(c) A. Clozapine is referred as an "atypical" antipsychotic agent. It can cause agranulocytosis
and should be monitored with weekly CBC. This is not a first line treatment drug.
(u) B. See A for explanation.
(u) C. See A for explanation.
(u) D. See A for explanation.
Ref: (10)

192. Clinical Therapeutics/Psychiatry/Behavioral Medicine


Which of the following is effective in the long-term treatment of panic attacks?
Answers
A. Triazolam (Halcion)
B. Propranolol (Inderal)
C. Lorazepam (Ativan)
D. Fluoxetine (Prozac)
Explanations
(u) A. Triazolam is a benzodiazepine used for sleep.
(u) B. Propranolol blocks the peripheral symptoms associated with panic attacks (palpitations,
tremors). It is not effective for treating the underlying disorder.
(u) C. Lorazepam may be used for urgent treatment of panic disorders, but should not be used for
long-term treatment.
(c) D. SSRI's, such as fluoxetine, are the initial drugs of choice for the long-term treatment of
panic disorders.

87

Ref: (27)

193. Clinical Therapeutics/Psychiatry/Behavioral Medicine


A child has been under treatment for attention-deficit hyperactivity disorder (ADHD). No
response has occurred with behavioral adaptations. Which of the following categories of
medication should this patient be given?
Answers
A. stimulants
B. anxiolytics
C. antipsychotics
D. antidepressants
Explanations
(c) A. Stimulants, such as Ritalin, Dexedrine, and Cylert, are effective in 50 to 80% of children
with ADHD.
(u) B. Anxiolytic agents, such as Xanax, are used in treating anxiety disorders.
(u) C. Antipsychotics, such as Thorazine, are used in treating psychosis.
(u) D. Antidepressants, such as Elavil, are used to treat depression.
Ref: (14)

194. Clinical Therapeutics/Pulmonology


A 36 year-old male developed a sore throat and was treated with IM penicillin. Within 20
minutes, he felt faint, became dyspneic, and had diarrhea. Upon entry to the emergency
department, he was pale and apprehensive. He had a thready pulse, and systolic blood
pressure was 40 mmHg. Which of the following is the most appropriate initial agent to
use?
Answers
A. Dopamine
B. Epinephrine
C. Hydrocortisone
D. Diphenhydramine
Explanations
(u) A. Dopamine is not indicated in the treatment of allergic reactions.
(c) B. Epinephrine is the drug of first choice for emergency use and should be given as soon as
anaphylactic shock is suspected or diagnosed.
(u) C. Hydrocortisone should be given as an adjunct to epinephrine, but it is not the drug of first
choice.
(u) D. Diphenhydramine should be given as an adjunct to treatment, but it is not the drug of first
choice.
Ref: (28)

195. Clinical Therapeutics/Pulmonology


A 22 year-old female with a history of asthma presents with complaints of increasing
asthma attacks. The patient states she has been well controlled on albuterol inhaler
until one month ago. Since that time she notices that she has had to use her inhaler 3-4
times a week and also has had increasing nighttime use averaging about three episodes in

88

the past month. Spirometry reveals > 85% predicted value. Which of the following is the
most appropriate intervention at this time?
Answers
A. Oral prednisone
B. Oral theophylline
C. Salmeterol inhaler
D. Beclomethasone inhaler
Explanations
(u) A. Oral corticosteroids, such as prednisone, are added to therapy in severe persistent asthma.
While a course of oral corticosteroids may be needed for mild exacerbations of asthma, they are
not added until inhaled corticosteroids have failed to control the symptoms.
(u) B. Due to its safety profile, oral theophylline is now considered a third or fourth line treatment
option for asthma.
(u) C. Long acting inhaled beta2-agonists, such as salmeterol, are not added to the treatment
regimen until the symptoms indicate a moderate persistent asthma. Long acting inhaled beta2
agonists should also not be used in place of inhaled steroids.
(c) D. This patient has progressed to mild persistent asthma. In addition to her inhaled beta2agonist (albuterol), she should be started on an anti-inflammatory agent. Inhaled corticosteroids,
such as beclomethasone, are preferred for long-term control. Other options may include
cromolyn or nedocromil.
Ref: (27)

196. Clinical Therapeutics/Pulmonology


A 42 year-old male with unremarkable past medical history is admitted to the general
medical ward with community-acquired pneumonia. He has a 20 pack-year history of
cigarette smoking. He is empirically started on ceftriaxone (Rocephin). Which of the
following antibiotics would be most appropriate to add to his empiric treatment regimen?
Answers
A. Piperacillin
B. Vancomycin
C. Clindamycin
D. Azithromycin
Explanations
(u) A. Antipseudomonal penicillins, such as piperacillin, added to an aminoglycoside or
fluoroquinolone are indicated for empiric treatment of nosocomial pneumonia.
(u) B. Use of vancomycin should be restricted to cases where there is a high index of suspicion of
involvement of methicillin resistant <i>Staphylococcus aureus</i> (MRSA).
(u) C. Clindamycin is indicated in cases of suspected anaerobe involvement.
(c) D. Patients with community-acquired pneumonia who require hospitalization on the general
medical ward are treated with an extended-spectrum beta-lactam antibiotic, such as ceftriaxone,
with a macrolide, such as azithromycin. Addition of a macrolide is also recommended due to the
patients smoking history and possible involvement of <i>Hemophilus influenzae</i>.
Ref: (27)

197. Clinical Therapeutics/Pulmonology


A 40 year-old alcoholic male presents with sudden onset of severe chills, fever, dyspnea
and cough productive of red mucoid sticky sputum. He appears ill looking with cyanosis.
Examination reveals vital signs: Temp 102 degrees F; Pulse 120 /minute and regular;

89

RR 28/min; BP 90/62 mm Hg. Lungs reveal minimal rales and dullness in the right upper
lobe with decreased breath sounds. Chest x-ray reveals right upper lobe consolidation
with a bulging fissure. Gram stain reveals many white blood cells and many gramnegative rods. Which of the following is the most appropriate drug of choice?
Answers
A. Ticarcillin
B. Cefotaxime
C. Doxycycline
D. Clindamycin
Explanations
(u) A. Infections caused by Klebsiella organisms are resistant to both ticarcillin and ampicillin.
(c) B. This patient most likely has pneumonia caused by Klebsiella. A third generation
cephalosporin, such as cefotaxime, is the preferred antimicrobial therapy against <i>Klebsiella
pneumoniae</i>. Alternative antibiotic choices may include a carbapenem, beta-lactam/betalactamase inhibitor or a fluoroquinolone.
(u) C. See B for explanation.
(u) D. See B for explanation.
Ref: (27)

198. Clinical Therapeutics/Pulmonology


Which of the following is the greatest risk factor for the development of drug resistance in
the treatment of tuberculosis?
Answers
A. A child with cystic fibrosis
B. An elderly patient in a nursing home
C. Non-adherence to prescribed drug regimen
D. Patients with a history of diabetes mellitus
Explanations
(u) A. Children with cystic fibrosis are at an increased risk for various lung infections, but not drug
resistance.
(u) B. While institutionalized patients, such as nursing home residents, are at increased risk for
infection with <i>Mycobacterium tuberculosis</i>, the patient is not at increased risk for drug
resistance.
(c) C. Non-adherence to prescribed drug regimens is a risk factor for the development of drug
resistance as well as being a major cause of treatment failure and continued transmission of
tuberculosis. Patient education about the importance of drug therapy is important in an attempt to
avoid this.
(u) D. Patients with a history of diabetes mellitus are at increased risk for active disease, not drug
resistance.
Ref: (27)

199. Clinical Therapeutics/Urology/Renal


Counseling a patient with erectile dysfunction who is taking sildenafil (Viagra) should
include warning of
Answers
A. developing hypotension.
B. urinary retention.

90

C. development of constipation.
D. an increased risk of prostate cancer.
Explanations
(c) A. Sildenafil is a vasodilator, which can result in a decrease in preload leading to hypotension.
(u) B. Urinary retention is not a side effect of sildenafil.
(u) C. Constipation does not occur with sildenafil, but diarrhea may occur.
(u) D. Sildenafil is not a hormone and does not increase the risk of developing prostate cancer.
Ref: (10)

200. Clinical Therapeutics/Urology/Renal


A 34 year-old male presents with symptoms of painful urethral discharge. History reveals
recent (7 days ago) intercourse with a new partner. A Gram stain is negative for
intracellular diplococci. Assuming no allergies, which of the following is the antibiotic of
choice?
Answers
A. Doxycycline
B. Penicillin G
C. Ciprofloxacin (Cipro)
D. Cephalexin (Keflex)
Explanations
(c) A. Tetracyclines, such as doxycycline, are drugs of first choice for Chlamydia.
(u) B. Penicillin G is not used in treating chlamydial infections.
(u) C. Cephalexin and ciprofloxacin are not effective in treating Chlamydial infections.
(u) D. See C for explanation.
Ref: (32)

201. Clinical Therapeutics/Urology/Renal


A 7-year-old boy wets the bed nearly every night. Which of the following is the best
pharmaceutical agent to use in treating this patient?
Answers
A. Desmopressin (DDAVP)
B. Paroxetine (Paxil)
C. Lorazepam (Ativan)
D. Hyoscyamine (Levsin)
Explanations
(c) A. Desmopressin, while not curative, will relieve symptoms.
(u) B. Paroxetine is not indicated for enuresis.
(u) C. Lorazepam is used for treatment of sleep terrors.
(u) D. Hyoscyamine is used in the treatment of pediatric overactive bladder.
Ref: (6)

202. Clinical Therapeutics/Urology/Renal


Of the following, which is the appropriate management of a pregnant patient with
asymptomatic bacteriuria?

91

Answers
A. Trimethoprim-sulfamethoxazole (Septra)
B. Tetracycline
C. Nitrofurantoin (Macrobid)
D. No treatment is needed
Explanations
(u) A. Trimethoprim-sulfamethoxazole is not approved for pregnancy.
(h) B. Tetracycline is not indicated in pregnancy as it can cause teeth discoloration and inhibition
of fetal skeletal growth.
(c) C. Nitrofurantoin is safe and preferred treatment for urinary tract infections in pregnancy.
(h) D. Asymptomatic bacteriuria should be treated in pregnancy to prevent development of
complications, such as premature labor, fetal morbidity, and premature mortality.
Ref: (8)

203. Clinical Therapeutics/Infectious Diseases


Which of the following is the treatment of choice for Influenzae B?
Answers
A. Amantadine (Symmetrel)
B. Zanamivir (Relenza)
C. Acyclovir (Zovirax)
D. Nevirapine (Viramune)
Explanations
(u) A. Amantadine is only used in the treatment of Influenzae A.
(c) B. Zanamivir is used to treat both Influenzae A and B.
(u) C. Acyclovir is used to treat viral infections due to certain herpes viruses but not influenzae.
(u) D. Nevirapine is used to treat infection due to HIV.
Ref: (8)

204. Scientific Concepts/Cardiology


Which of the following is the mechanism of action of Class III antiarrhythmic drugs?
Answers
A. Na+ channel blocker
B. K+ channel blocker
C. Beta adrenoreceptor blocker
D. Ca++ channel blocker
Explanations
(u) A. Na+ channel blockers are Class I.
(c) B. K+ channel blockers are Class III.
(u) C. Beta adrenoreceptor blockers are Class II.
(u) D. Ca++ channel blockers are Class VI.
Ref: (27)

205. Scientific Concepts/Cardiology


In congestive heart failure the mechanism responsible for the production of an S3 gallop
is

92

Answers
A. contraction of atria in late diastole against a stiffened ventricle.
B. rapid ventricular filling during early diastole.
C. vibration of a partially closed mitral valve during mid to late diastole.
D. secondary to closure of the mitral valve leaflets during systole.
Explanations
(u) A. Atrial contraction against a noncompliant ventricle is the mechanism responsible for S4.
(c) B. Rapid ventricular filling during early diastole is the mechanism responsible for the S3.
(u) C. Vibration of a partially closed mitral valve during mid to late diastole is the mechanism
responsible for the Austin-Flint murmur of aortic regurgitation.
(u) D. Closure of the mitral valve leaflets during systole is the mechanism responsible for part of
the S1 heart sound.
Ref: (8)

206. Scientific Concepts/Cardiology


What is the most likely mechanism responsible for retinal hemorrhages and neurologic
complications in a patient with infective endocarditis?
Answers
A. Metabolic acidosis
B. Systemic arterial embolization of vegetations
C. Hypotension and tachycardia
D. Activation of the immune system
Explanations
(u) A. See B for explanation.
(c) B. The vegetations that occur during infective endocarditis can become emboli and can be
dispersed throughout the arterial system.
(u) C. See B for explanation.
(u) D. Glomerulonephritis and arthritis result from activation of the immune system.
Ref: (8)
207. Scientific Concepts/Cardiology
During an inferior wall myocardial infarction the signs and symptoms of nausea and
vomiting, weakness and sinus bradycardia are a result of what mechanism?
Answers
A. Increased sympathetic tone
B. Increased vagal tone
C. Activation of the renin-angiotensin system
D. Activation of the inflammatory and complement cascade system
Explanations
(u) A. See B for explanation.
(c) B. Increased vagal tone is common in inferior wall MI; if the SA node is involved, bradycardia
may develop.
(u) C. See B for explanation.
(u) D. See B for explanation.
Ref: (8)

93

208. Scientific Concepts/Dermatology


Which of the following is characterized by epidermal hyperplasia and an increase in the
epidermal turnover?
Answers
A. Atopic dermatitis
B. Tinea corporis
C. Ecthyma
D. Psoriasis
Explanations
(u) A. Inflammation in atopic dermatitis results in lichenification of the skin from itching, which
leaves the skin dry and scaly when it resolves. The inflammation is the result of elevated Tlymphocyte activation, defective cell-mediated immunity, and IgE overproduction. Epidermal
hyperplasia is not involved in atopic dermatitis.
(u) B. Tinea corporis is a fungal infection and is does not have epidermal hyperplasia.
(u) C. Erythema is the result of an infection from group A beta-hemolytic streptococcus and
contaminated with staphylococci. Like atopic dermatitis and tinea corporis, ecthyma has no
epidermal hyperplasia.
(c) D. Psoriasis is characterized by an increased epidermal cell turnover, increased numbers of
epidermal stem cells, and an abnormal differentiation of keratin . This leads to the classic scale
associated with psoriasis.
Ref: (11)

209. Scientific Concepts/Endocrinology


Which of the following conditions may result in hypokalemia?
Answers
A. Adrenal adenoma
B. Hypoparathyroidism
C. Hyperthyroidism
D. Adrenal insufficiency
Explanations
(c) A. Excessive secretion of aldosterone from an adrenal adenoma will lead to sodium retention
and the secretion of potassium in the distal tubule of the kidney, eventually leading to
hypokalemia.
(u) B. Hypoparathyroidism and hyperthyroidism should not have any affect on potassium levels.
(u) C. See B for explanation.
(u) D. Adrenal insufficiency would lead to hyperkalemia.
Ref: (8)

210. Scientific Concepts/ENT/Ophthalmology


A 53 year-old woman complains that she has not been able to smell for several weeks.
Prior to an upper respiratory infection 3 weeks ago, her sense of smell was "just fine."
The most likely cause of the anosmia is
Answers
A. intranasal obstruction.
B. destruction of the olfactory neuroepithelium.

94

C. thickened mucus covering the olfactory cilia.


D. depletion of the G-protein in the ciliary membrane.
Explanations
(u) A. Intranasal obstruction is common with acute bacterial or viral infections, but anosmia
resolves when the obstruction resolves.
(c) B. The olfactory epithelium is destroyed by viral infections and chronic rhinitis.
(u) C. Thickened mucus does not inhibit odorants from reaching the neuroepithelium.
(u) D. G-protein is not depleted in viral URIs.
Ref: (8)

211. Scientific Concepts/ENT/Ophthalmology


Which of the following does the macula provide?
Answers
A. Night vision
B. Color vision
C. Peripheral vision
D. Central vision acuity
Explanations
(u) A. Night vision is a function of rod photoreceptors, which are found in the peripheral retina.
(u) B. Color vision is a function of cone photoreceptors.
(u) C. The peripheral retina is responsible for peripheral vision.
(c) D. The macula is responsible for central visual acuity.
Ref: (8)

212. Scientific Concepts/Gastrointestinal/Nutritional


Which of the following describes the pathophysiology of pancreatitis related to heavy
alcohol intake?
Answers
A. Alcohol increases the concentration of protein in the pancreatic juices and this protein can
precipitate in the pancreatic ducts and produce obstructive plugs.
B. Alcohol increases the level of triglycerides and this results in increased concentration of
pancreatic enzymes.
C. Alcohol paralyzes the sphincter of Oddi, which interferes with the ability of the pancreas to
release the enzymes lipase and amylase.
D. Alcohol alters the concentration of bile salts, which results in the formation of gallstones, which
results in pancreatitis.
Explanations
(c) A. This is the explanation as to why alcohol results in acute pancreatitis. Trapped within the
gland, the proteolytic enzymes trypsin and chymotrypsin digest pancreatic tissue that digests
pancreatic tissue.
(u) B. See A for explanation.
(u) C. See A for explanation.
(u) D. See A for explanation.
Ref: (8)

95

213. Scientific Concepts/Gastrointestinal/Nutritional


Hepatitis D infection (HDV) or delta agent hepatitis requires co-infection with which of the
following other types of hepatitis?
Answers
A. HAV
B. HBV
C. HCV
D. HEV
Explanations
(u) A. See B for explanation.
(c) B. HDV is a defective RNA virus that requires simultaneous or antecedent HBV infection in
order to become an active pathogen. Coinfection with HBV and HDV produces a fulminant acute
hepatitis.
(u) C. See B for explanation.
(u) D. See B for explanation.
Ref: (8)

214. Scientific Concepts/Hematology


Pernicious anemia is due to deficiency in which of the following?
Answers
A. Cobalamin-binding proteins
B. Pancreatic enzymes
C. Spectrin
D. Intrinsic factor
Explanations
(u) A. Cobalamin-binding proteins and pancreatic enzyme deficiencies are causes of vitamin B12
deficiency, but not pernicious anemia.
(u) B. See A for explanation.
(u) C. Abnormalities in spectrin and actin are noted in hereditary spherocytosis.
(c) D. Pernicious anemia is caused by lack of intrinsic factor.
Ref: (8)

215. Scientific Concepts/Neurology


A 58 year-old truck driver is found to have a positive Romberg test and loss of vibratory
sensation in his lower extremities. What anatomical structure is the likely affected?
Answers
A. Cerebellum
B. Posterior column
C. Sensory cortex
D. Vestibular apparatus
Explanations
(u) A. See B for explanation.
(c) B. Posterior column is responsible for vibratory sensation and proprioception. Romberg test is
a general test of proprioception with disease of the cerebellum, vestibular apparatus or posterior
column being the most likely source.
(u) C. The bilateral lower extremity findings make a lesion of the sensory cortex unlikely.

96

(u) D. See B for explanation.


Ref: (18)

216. Scientific Concepts/Obstetrics/Gynecology


Which of the following autosomal disorders results from the chromosomal abnormality of
trisomy 21?
Answers
A. Edward's syndrome
B. Down's syndrome
C. Wolf's syndrome
D. Cri du chat syndrome
Explanations
(u) A. Edwards syndrome is due to trisomy 18.
(c) B. Downs syndrome is due to trisomy 21
(u) C. Wolfs syndrome is due to deletion of the short arm chromosome 4.
(u) D. Cri du chat syndrome is due to deletion of the short arm chromosome 5.
Ref: (9)

217. Scientific Concepts/Obstetrics/Gynecology


Human papilloma virus has been strongly associated with the development of which of the
following?
Answers
A. Cervical carcinoma
B. Ovarian carcinoma
C. Pelvic inflammatory disease
D. Vaginitis
Explanations
(c) A. Human papilloma virus (types 16, 18, and 31) has been linked to the development of
cervical carcinoma.
(u) B. See A for explanation.
(u) C. See A for explanation.
(u) D. See A for explanation.
Ref: (8)

218. Scientific Concepts/Orthopedics/Rheumatology


Why are fractures of the scaphoid bone prone to aseptic necrosis?
Answers
A. Disrupted arterial supply
B. Lack of weight bearing
C. Inadequate immobilization
D. Impaired venous drainage
Explanations
(c) A. Scaphoid bone blood supply is from one single vessel that enters distally and once
disrupted causes aseptic necrosis.

97

(u) B. Weightbearing does aid in callous formation, but the poor proximal blood supply of the
scaphoid is the primary cause of increased aseptic necrosis.
(u) C. Short-arm thumb spica casting adequately immobilizes the wrist.
(u) D. Injury is to the arterial supply and not to venous drainage.
Ref: (25)

219. Scientific Concepts/Orthopedics/Rheumatology


An elderly patient presents to the ED complaining of right shoulder pain occurring after
falling on his right outstretched hand with the elbow extended. On physical examination
the arm is pale and pulseless and there is tenderness of the upper arm. Which of the
following structures is most likely injured?
Answers
A. Axillary artery
B. Subclavian vein
C. Superior thoracic artery
D. Ulnar vein
Explanations
(c) A. Injury to the axillary artery is the most common vascular injury in proximal fractures of the
humerus due to the close approximation of the axillary artery to the coracoid process.
(u) B. See A for explanation.
(u) C. See A for explanation.
(u) D. See A for explanation.
Ref: (28)

220. Scientific Concepts/Psychiatry/Behavioral Medicine


Phenothiazines exert their antipsychotic effects by blocking
Answers
A. alpha-adrenergic receptors.
B. dopamine receptors.
C. histamine receptors.
D. serotonin receptors.
Explanations
(u) A. Phenothiazine blockage of alpha-adrenergic and histamine receptors lead to orthostatic
hypotension, sedation, and anticholinergic effects.
(c) B. Blockage of the dopamine receptors in the mesolimbic areas of the brain is responsible for
the antipsychotic effects of the phenothiazines; blockage of the dopamine receptors in the
nigrostriatal areas lead to the motor side effects, such as dystonia and akathisia.
(u) C. See A for explanation.
(u) D. Phenothiazines do not block serotonin receptors.
Ref: (10)

221. Scientific Concepts/Pulmonology


Which of the following has been found to be an important reservoir in the transmission of
pertussis?
Answers

98

A. Mosquitoes
B. Adult humans
C. Dogs and cats
D. White-tailed deer
Explanations
(u) A. While mosquitoes have been implemented in the spread of several infectious diseases,
pertussis is not one of them.
(c) B. Since neither the vaccine nor the disease itself provides lasting immunity against pertussis,
adult humans are an important reservoir in the transmission of pertussis.
(u) C. Contact with dogs and cats has lead to the development of upper and lower respiratory
infections that are caused by <i>Bordetella bronchiseptica</i>, but not pertussis.
(u) D. White-tailed deer are part of the transmission cycle for Lyme disease, not pertussis.
Ref: (27)

222. Scientific Concepts/Pulmonology


Which of the following best describes the pathophysiology of emphysema?
Answers
A. Interstitial inflammation and fibrosis
B. Alveoli enlargement and loss of septa
C. Mucosal edema and inflammatory response
D. Excessive mucus secretion and chronic cough
Explanations
(u) A. Interstitial inflammation and fibrosis are seen with restrictive causes of lung disease, such
as asbestosis.
(c) B. Emphysema results from alveoli enlargement with loss of septal wall integrity without any
evidence of fibrosis.
(u) C. Mucosal edema and inflammatory response are seen with asthma.
(u) D. Excessive mucus secretion and chronic cough are characteristic of chronic bronchitis.
Ref: (27)

223. Scientific Concepts/Pulmonology


Which of the following mechanisms leads to a primary pneumothorax?
Answers
A. Penetrating or blunt trauma forces
B. Underlying lung cancer
C. Pressure of air in the pleural space exceeds room air pressure
D. Rupture of subpleural apical blebs due to high negative intrapleural pressures
Explanations
(u) A. Penetrating or blunt trauma force is responsible for a traumatic pneumothorax.
(u) B. A pneumothorax that results from an underlying lung disease is classified as a secondary
pneumothorax.
(u) C. When pressure of air in the pleural space exceeds room air pressure, it leads to a tension
pneumothorax.
(c) D. A primary spontaneous pneumothorax is thought to result from a rupture of subpleural
apical blebs secondary to high negative intrapleural pressures.
Ref: (27)

99

224. Scientific Concepts/Urology/Renal


A male patient presents with hematuria. Upon further questioning the patient states that
the hematuria occurs at the end of his urinary stream. Which of the following is the most
likely source of blood?
Answers
A. Renal pelvis
B. Bladder neck
C. Anterior urethra
D. Ureter
Explanations
(u) A. Total hematuria, blood throughout the urinary stream, suggests a bladder or upper urinary
tract source.
(c) B. Terminal hematuria, blood at the end of the urinary stream, suggests a bladder neck or
prostatic urethral source.
(u) C. Presence of blood at the beginning of the urinary stream suggests an anterior (penile)
urethral source.
(u) D. Hematuria from the kidneys or ureter may be present microscopically or throughout the
stream.
Ref: (27)

225. Scientific Concepts/Infectious Diseases


A 29 year-old construction worker presents with cough, fever, and malaise, two weeks
after demolishing an old chicken coop. Chest x-ray reveals hilar adenopathy. Gram stain
of the sputum reveals many oval, budding cells. Which of the following is the most likely
infectious agent?
Answers
A. Cryptococcus neoformans
B. Mycoplasma pneumoniae
C. Streptococcus pneumoniae
D. Histoplasma capsulatum
Explanations
(u) A. Cryptococcus neoformans pulmonary disease is typically asymptomatic and hilar
adenopathy is rare. Gram stain will reveal budding yeast with pseudohyphae.
(u) B. Mycoplasma pneumoniae is not found in bird droppings. Chest x-ray typically reveals an
interstitial infiltrate. Mycoplasma will not be seen on Gram stain.
(u) C. Streptococcus pneumoniae presents with fever and cough. Air-space consolidation is
predominant on chest x-ray. On Gram stain, gram-positive cocci will be noted.
(c) D. Histoplasma capsulatum is typically found in bird droppings in old buildings and chicken
coops. The organism is inhaled and causes high fever, cough, and general malaise. Chest x-ray
reveals hilar adenopathy. Gram stain reveals budding yeast.
Ref: (8)

100

Association of Physician Assistant Programs


PACKRAT

Comment Form
Please fill out this form and return it to: APAP
950 N. Washington St.
Alexandria, VA 22314
YES

NO

1. I found the PACKRAT a useful study tool.


Comment:

2. I was able to understand the directions in the Directions and Explanations.

Comment:

3. The feedback provided was easy to understand.


Comment:

4. I know where my weaknesses lie and where to focus my studies.


Comment:

5. The content of the examination was comprehensive.


Comment:

101

6. The examination questions were clearly worded.


Comment:

7. Suggestions for improvement:


_________________________________________________________________
_________________________________________________________________

102

You might also like